Консультация онколога



бет1/55
Дата01.06.2022
өлшемі2,71 Mb.
#145722
  1   2   3   4   5   6   7   8   9   ...   55
Байланысты:
ктиц
Психология-2 РК



ИГА русс 2021-2022
Мужчина 36 лет. Строитель, работа связана с пребыванием на открытом воздухе, под солнцем. При профилактическом осмотре врач обратил внимание на невус (фото). Со слов пациента родинка стала увеличиваться в размере в последние месяцы. Какая дальнейшая тактика наиболее целесообразна?
консультация онколога
наблюдение, это пограничный невус
консультация дерматолога
определение уровня белка S100 до консультации онколога
биопсия невуса до консультации онколога
Больной 48 лет с диагнозом цирроз печени в исходе вирусного гепатита С, компенсированный. Какое исследование следует проводить больному каждые 3 месяца с целью раннего выявления возможного онкологического осложнения?
АФП и УЗИ печени и селезенки
МРТ печени и селезенки
пункционная биопсия печени
ФЭГДС
СЕА и СА19-9
13-летний мальчик с жалобами на прогрессирующую боль в левой ноге в течение 2-х месяцев, которая мешает ему даже спать. Мама давала ему ибупрофен, но боли не прошли. За неделю до того, как началась боль, пациенту ударили по бедру мячом, в результате чего нога покраснела и опухла в течение нескольких дней. В ОАК лейкоциты – 21 000/мм3, СОЭ 68 мм/час. На рентгене – множественные литические поражения в средней трети бедренной кости.
Какой диагноз наиболее вероятен?
Саркома Юинга
Остеосаркома
Хондросаркома
Остеохондрома
Хондробластома
Женщина 45 лет. Не замужем, в анамнезе 3 беременности, все - искусственное прерывание беременности. Курит в течение 28 лет. Положенный скрининг не проходила. При пальпации молочных желез заметила небольшое уплотнение в наружном верхнем квадранте. Какая дальнейшая тактика?
выполнить маммографию
направить на консультацию к гинекологу
направить на консультацию к маммологу
КТ органов грудной клетки
биопсия уплотненного участка
Женщина 65 лет. Обратилась с жалобами на увеличение живота, недомогание слабость. УЗИ выявило свободную жидкость в брюшной полости в объеме более 1,5 л. Размеры печени и селезнки в пределах нормы, диаметр v.portae - 1,0 см. Онкомаркеры - ОММА (СА-125) резко повышен, альфа-фетопротеин - повышен в 3 раза. Какой предварительный диагноз наиболее вероятен?
рак яичников
гепатоцеллюлярная карцинома
рак поджелудочной железы
эндометриоидная киста
эндометриоз
Женщина 72 лет, в анамнезе инфаркт миокарда, сахарный диабет. В анализах: холестерин 5,97 ммоль/л, ЛПНП 4,53 ммоль/л, ЛПВП 0,98 ммоль/л, триглицериды 2,24 ммоль/л, коэффициент атерогенности 5,09. Рассчитанный 10-летний показатель SCORE ≥10%
Врач назначил аторвастатин. До какого целевого уровня необходимо снижение уровня ЛПНП (УД – I B) согласно протоколу МЗ РК «Артериальная гипертензия» от 03.10.2019 г.?
<1,8 ммоль/л
<1,9 ммоль/л
<2.0 ммоль/л
<2,5 ммоль/л
<2,8 ммоль/л
Девушка 14 лет, страдающая ювенильным ревматоидным артритом, получает метотрексат. Мать девушки обеспокоена тем, что у дочери может быть повышен риск развития онкологической патологии. В семье девушки тетя умерла от рака шейки матки. Какое высказывание ВЕРНО в отношении развития рака шейки матки в данной ситуации?
Американская ассоциация ревматологов рекомендует данной категории пациентов вакцинацию от ВПЧ четырехвалентной вакциной для снижения риска рака шейки матки
Риск развития рака у данной категории больных значительно выше, чем в популяции
Риск развития рака у данной категории больных значительно ниже, чем в популяции благодаря терапии метотрексатом
Риск развития рака у ревматологических больных не выше, чем в популяции, но чаще развиваются лимфомы
Риск развития рака у ревматологических больных не выше, чем в популяции, но чаще развиваются доброкачественные опухоли
Мужчина 72 лет. Беспокоят упорные по нескольку дней запоры. Похудел за последние 3 месяца на 5 кг. В крови - панцитопения и СОЭ 65 мм рт.ст. Какое обследование необходимо провести для уточнения диагноза?
колоноскопия с прицельной биопсией
пальпаторное исследование прямой кишки
Альфа-фетопротеин
КТ органов брюшной полости
Ирригосокпия
Мужчина 62 лет, в анамнезе инсульт, сахарный диабет. 10-летний риск по шкале SCORE≥10%. Принимает антигипертензивную терапию. Участвует в программе управления заболеваниями. Какая периодичность осмотров врачом ПМСП рекомендована этому пациенту согласно «Правил оказания первичной медико-санитарной помощи и Правил прикрепления к организациям первичной медико-санитарной помощи» (приказ № 281 от 28 апреля 2015 г)?
1 раз в 3 месяца
1 раз в 1 месяца
1 раз в 2 месяца
1 раз в 6 месяцев
1 раз в 12 месяцев
Женщина 65 лет, в анамнезе АГ, ИБС, аортокоронарное шунтирование, сахарный диабет. Принимает терапию. В анализах - Глюкоза 153.7 mg/dL, Мочевина - 53,62 mg/dL, Креатинин 1,08 mg/dL, Мочевая к-та - 314,2 µmol/L, билирубин общ 0,4 mg/dL, билирубин прямой - 0,06 mg/dL, ALT 19,82 U/L, AST 19,18 U/L, С-реакт. белок 0,17 mg/dL, холестерин общ - 5,91 mmol/L, липопротеины высокой плотности - 1,07, липопротеины низкой плотности - 4,28, триглицериды - 2,85 mg/dL. Врач назначил аторвастатин. Укажите рекомендуемые показатели функции печени (АЛТ) при приеме статинов? 
не должны превышать >3 ВГН
не должны превышать >4 ВГН
не должны превышать >5 ВГН
не должны превышать >6 ВГН
не должны превышать >7 ВГН
Мужчина 57 лет, страдает артериальной гипертензией. Принимает антигипертензивную терапию. На приеме спросил врача по поводу употребления алкоголя.
Укажите рекомендуемое ограничение употребления алкоголя для мужчин (УД – IA)?
менее 14 ед/нед
менее 16 ед/нед
менее 18 ед/нед
менее 20 ед/нед
менее 22 ед/нед
Женщина 52 лет, страдает артериальной гипертензией. Принимает антигипертензивную терапию. На приеме спросила врача по поводу употребления алкоголя.
Укажите рекомендуемое ограничение употребления алкоголя для женщин (УД – IA)?
менее 8 ед/нед
менее 10 ед/нед
менее 12 ед/нед
менее 14 ед/нед
менее 16 ед/нед
Женщина 36 лет, страдающая артериальной гипертензией, длительно получала антигипертензивный препарат. Но в связи с беременностью 3-4 недель, кардиолог учитывая тератогенный эффект отменила его. Какой из перечисленных препаратов НАИБОЛЕЕ вероятно обладает этим эффектом?
Периндоприл
Клонидин
Метилдопа
Нифедипин
Метопролол
При обследовании у беременной пациентки были выявлены антитела к ВПГ-1, ЦМВ и ВЭБ (в ИФА ОП образцов в 2 раза выше ОП критической). Герпетические высыпания отрицает, жалоб нет, лимфаденопатии не выявлено. Какая тактика целесообразна в данной ситуации?
Наблюдение
Порекомендовать превентивный курс ацикловира
Провести курс индуктора интерферона
Провести курс ганцикловира
Назначить арбидол
Третья (III) степень стойких нарушений функций организма – это выраженные нарушения в диапазоне?
70; 80
70; 85
60; 70
60; 80
60; 75
Мужчина 22 лет, на профосмотре артериальное давление 150/80 мм рт, на обеих руках. Все жизненные показатели в норме. Какая тактика наиболее целесообразна?
модификация образа жизни
1 ступень (начальная терапия) — двойная комбинация
2 ступень — тройная комбинация
3 ступень — тройная комбинация + спиронолактон или другой препарат
монотерапия
79-летний мужчина обратился с жалобами на боли в нижней части живота в течение 1 часа, в анамнезе рак простаты. Он не мочился 24 часа. При осмотре – выявлен пальпируемый мочевой пузырь, чувствительный к пальпации. УЗИ – острая задержка мочи. Трансуретральная катетеризация безуспешна. Уролог планирует сделать надлобковую катетеризацию, но сейчас находится у другого пациента. Уролог попросил резидента, который никогда не делал катетеризацию, взять у пациента информированное согласие на процедуру. Какое из следующих утверждений резидента является наиболее подходящим?
«Я бы предпочел, чтобы вы получили информированное согласие, когда вы сами освободитесь»
«Я сделаю так, чтобы пациент прочитал и подписал форму информированного согласия»
«Я был бы рад получить информированное согласие от вашего имени, но мне не разрешено делать это по закону»
«Надлобковая катетеризация не является выбором для этого пациента»
«Я попрошу пациента отказаться от информированного согласия, потому что это срочная процедура»
Женщина 24 лет с вирусным гепатитом В, беременность 32 недели. Биохимия: АЛТ – 0,64 ммоль/л. АСТ – 0,47 ммоль/л. Билирубин – 22,25 мкмоль/л. HBsAg – положительный, anti HBs – отр., antiHBc IgM – положит., antiHBc IgG – положит., HBeAg – положит., anti HBe – отр.;. ПЦР HBV ДНК – 106 копий/мл. Какая тактика наиболее целесообразна для снижения риска перинатальной передачи вируса ребенку?
Ламивудин 100 мг\сут до родов
Интерферон короткого действия
Интерферон в виде свечей до родов
Адеметионин+урсодезоксихолевая кислота
Эссенциальные фосфолипиды до 38-39 недели
Женщина 47 лет отмечает боли в области сердца жгучего характера, купируемые приемом валидола или седативных микстур. Часто возникает чувство жара, сопровождающееся гиперемией лица, потливостью. У женщины аменорея в течение 6 месяцев. При осмотре: кожные покровы влажные, гиперемированные. ЧСС-102 в мин, АД-145/85 мм рт. ст. Анаприлиновая проба положительна. Влагалищный мазок атрофический. Какой из перечисленных диагнозов НАИБОЛЕЕ вероятен?
Патологический климакс
ИБС. Стенокардия Принцметала
НЦД по гипертоническому типу
Артериальная гипертензия
ИБС. Стенокардия напряжения ФК III
21-летний студент колледжа обратился по поводу рвоты и болей в эпигастральной области в течение 2 дней. Боль иррадирует в спину. В анамнезе – атопический дерматит и тиреоидит Хашимото, принимает левотироксин. Не получал никаких прививок. Пьет по выходным 1-2 пива и изредка курит марихуану. Пациент выглядит взволнованным, пот на лице. Температура тела 37,9ᴼС, пульс 105 уд/мин, ЧДД 16/мин, АД 130/80 мм рт ст. Живот вздут, болезнен при пальпации в эпигастральной области, на коже – скопления желтых бляшек на поверхности туловища и разгибательных поверхностях конечностей. Hb – 15.2 г/л, Са+ 7.9 мг/дл Что является наиболее подходящим следующим шагом в оценке состояния?
Определить уровень липидов в сыворотке крови
Провести тест на потоотделение
Измерить сывороточный титр на IgM свиного грипа
Выполнить эзофагокастродуоденоскопию
Провести рентген брюшной полости в вертикальном положении пациента
Мужчина 40 лет обратился к врачу из-за болей в пояснице, которые постепенно усиливаются в течение последних 2 месяцев. Боль бывает ночью и не изменяется, если он меняет свое положение. По утрам в течение часа скованность в пояснице, которая улучшается в течения дня. В последние 3 месяца у него было 3 эпизода острой подагры, по поводу чего принимает аллпуринол. Жизненные показатели в пределах нормы. При осмотре ө уменьшение поясничного сгибания и болезненность в крстцовоөподвздошных суставах. На рентгене диагноз подтверждается. Назначен индометацин и программа упражнений. Через 6 недель пациент сообщает об отсутствии улучшений. Какой шаг наиболее подходит для ведения пациента перед дальнейшей фармакотерапией?
Тест Манту
Прекратить аллопуринол
Измерение креатинина
Обследование глаз
Полный анализ крови
Женщина 25 лет, беременность 12-13 недель. Жалобы: на частое болезненное мочеиспускание, боль над лобком, частые императивные позывы. Данные жалобы в течение 2-3 дней после переохлаждения. При обследовании ОАК без особенностей, в ОАМ – лейкоциы до 20-30 в п/зр, бактерии+++. Какой из перечисленных предварительных диагнозов является НАИБОЛЕЕ вероятным?
Инфекция мочевых путей
Острый пиелонефрит
Мочекаменная болезнь
Бессимптомная бактериурия
Тубулоинтерстициальный нефрит
Женщина 29 лет, беременность вторая, 9-10 недель. Обратилась с жалобами на тошноту и рвоту по утрам в течение последних двух недель. Для постановки на учет обратилась на сроке 6-7 недель, масса тела на тот момент - 74 кг. Сейчас вес - 68 кг. Объективно: состояние средней тяжести, сухость кожи, за время осмотра дважды были позывы на рвоту. Какая тактика наиболее целесообразна?
Немедленная госпитализация
Лечить амбулаторно, перорально солевые растворы
Лечить в условиях дневного стационара, инфузионно кристаллоидные растворы
Решение вопроса о немедленном родоразрешении
Консультация инфекциониста
Укажите размер взноса государства на обязательное медицинское страхование, подлежащее уплате в фонд ОСМС, установленный с 01 января 2020 года согласно Закону об обязательном медицинском страховании.
1.4%
2%
1.6%
1%
1.8%
Ребенку 6 лет, проведена туберкулиновая проба Манту с 2 ТЕ, результат - папула размером 15 мм, Диаскинтест положительный. После дообследования ребенка выставлен диагноз: «Гиперергическая реакция». Ребенок часто болеет простудными заболеваниями и бронхитом. Какая Ваша тактика наиболее целесообразна?
провести химиопрофилактику изониазидом 6 месяцев
провести химиопрофилактику этамбутолом и рифампицином 2 месяца
провести химиопрофилактику изониазидом и этамбутолом 2 месяца
провести химиопрофилактику изониазидом 3 месяца
провести лечение по 1 категории
Мужчина 38 лет, приехал из страны категории 1b в страну с карантином по COVID. Жалоб не предъявляет, симптомов нет. Результат тестирования на COVID-2019 отрицательный. После проведенных карантинных мероприятий врач посоветовал ему придерживаться режима самоизоляции дома. Во сколько раз эффективность карантинных мероприятий при коронаровирусной инфекции способствует снижению распространения инфекции согласно моделированию с использованием «дерева решения»?
в 9 раз
в 2 раза
в 4 раза
в 7 раз
в 10 раз
Женщина, 28 лет, после родов (ребенку 3 месяца) обратилась к врачу с жалобами на головную боль, недомогание, повышенную утомляемость, раздражительность, нарушение сна, светобоязнь, субфебрильную температуру 37,3оС. Больной себя считает в течение недели. Объективно: общее состояние ближе к удовлетворительному, периферические лимфоузлы не пальпируются, в легких везикулярное дыхание. Наблюдается ригидность затылочных мышц на 1,5 п/п. На обзорной рентгенограмме ОГК в нижней доле справа очаг Гона и кальцинаты в корнях справа. Какой из перечисленных методов обследования является важным для своевременной диагностики развившегося осложнения?
люмбальная пункция
биохимический анализ крови
продольная томография
общий анализ крови
КТ головного мозга
Мужчина 19 лет, обратился к терапевту с жалобами на увеличение шейных лимфоузлов, их болезненность, повышение температуры до 38оС, слабость. При осмотре кожа над поверхностью лимфлоузлов гиперемирована. При пальпации лимфоузлы эластичны, подвижны, не спаяны между собой, болезненны. На рентгенограмме ОГК петрификаты в корнях. Какой из перечисленных методов обследования является наиболее информативным?
Биопсия лимфоузлов
рентгенография лимфоузлов
туберкулинодиагностика
трепанобиопсия костного мозга
КТ органов грудной клетки
Женщина 43 лет, приехала из страны категории 1b в страну с карантином по COVID. В самолете сидела на расстоянии 2 сидений от пассажира, у которого выявили COVID-2019. Жалоб не предъявляет, симптомов нет. Предварительный результат тестирования на COVID-2019 отрицательный. В течение какого времени рассматривается наличие эпидемиологической связи?
в течение 14 дней
в течение 10 дней
в течение 12 дней
в течение 16 дней
в течение 18 дней
Отец новорожденного ребенка болен фиброзно-кавернозным туберкулезом легких. Контакт больного отца с ребенком не желателен, но допустим:
через 6-8 недель после прививки БЦЖ
сразу после выписки из роддома, если ребенок привит БЦЖ
контакт не опасен при грудном вскармливании
через 9-10 недель после прививки БЦЖ
после излечения больного отца
Кто является плательщиками взносов в Фонд медицинского страхования?
Профессиональные медиаторы
Сотрудники специальных государственных органов
Сотрудники правоохранительных органов
Инвалиды
Руководители организаций
Мужчина 25 лет, с впервые выявленным инфильтративным туберкулезом S1 правого легкого, МБТ+ находился в стационаре 2 месяца, где проведена интенсивная фаза четырьмя ПТП. Далее ему должна была быть проведена 4-месячная поддерживающая фаза. Больной не придерживался рекомендованной схемы лечения. Бактериовыделение прекратилось. На рентгенограмме – в S1 правого легкого определятся четко округлая гомогенная тень, d – 3 см. Какой из перечисленных ниже диагнозов наиболее вероятен?
туберкулема
Склерозировавшийся очаг
периферический рак
заполненная киста
эхинококк
Женщина, 27 лет, после профилактического флюороосмотра вызвана на дообследование. Жалоб не предъявляет. В легких выслушивается везикулярное дыхание. Рентгенограмма: слева в верхней доле очаговые тени слабой интенсивности на фоне усиленного легочного рисунка. В анализе промывных вод бронхов МБТ не обнаружены. В анализе крови лейкоциты - 4,8 х 109, СОЭ – 4 мм/час. Какой из перечисленных ниже предварительных диагнозов наиболее вероятен?
очаговый туберкулез верхней доли левого легкого
инфильтративный туберкулез верхней доли левого легкого
карциноматоз верхней доли левого легкого
туберкулема верхней доли левого легкого
милиарный туберкулез легких
Мама ребенка 9 лет обратилась к врачу за разъяснениями по поводу медицинского страхования - надо ли ей платить за него взнос
До какого возраста дети освобождены от взносов в систему ОСМС?
До 18 лет
До 10 лет
До 12 лет
До 14 лет
До 16 лет
В чем суть принципа общественной солидарности при обязательном медицинском страховании?
Чем старше контингент застрахованного, тем больше взнос
Здоровый платит за больного
Пожилой платит за пожилого
Молодой платит за пожилого
Больной платит за больного
Женщина 38 лет обратилась на прием по поводу повышенного давления.
Как врач определит статус застрахованного гражданина?
Посредством введения ее ИНН
Посредством системы iKomek 109
Позвонив в единый call-центр 1414
Посредством EGOV
Позвонив в ФОМС
Новорожденный родился с желтухой, множественными геморрагическими проявлениями, ВПС в виде дефекта межжелудочковой перегородки, катарактой. Из анамнеза: мать ребенка во время беременности перенесла ОРВИ, аллергическая сыпь. Какие из ниже перечисленных возбудителей может быть наиболее вероятной причиной заболевания?
Вирус
Риккетсии
Хламидии
Грибы
Бактерии
У ребенка 5-ти лет отмечалось повышение температуры тела до 40оС, многократная рвота, головная боль, через 10 часов от начала заболевания появилась звездчатая, геморрагическая сыпь по телу, преимущественно в области ягодиц, голени, бедер. Наиболее вероятный диагноз:
менингококковая инфекция
стрептококковая инфекция
геморрагический васкулит
тромбоцитопеническая пурпура
иерсиниозная инфекция
Ребенок 3-х лет. Болен в течении 3-х дней. Начало с подъема Т тела, вялости, покашливания, насморка, конъюнктивита. На 4-й день все симптомы усилились, t 39оС, на лице появилась розовая, пятнисто-папулезная сыпь, которая на 5-й день распространилась на туловище, на следующий день и на конечности. За 10 дней до начала заболевания был контакт с родственником, у которого был кашель и «аллергическая сыпь». Наиболее вероятный предварительный диагноз
Корь
Скарлатина
герпетическая ангина
листериоз
ветряная оспа
Мужчина 79 лет, в анамнезе сахарный диабет 2 типа. Принимает валсартан 80 мг + гидрохлортиазид 12,5 мг. Достижение какого целевого уровня САД (УД - IA) рекомендовано этому пациенту согласно протоколу МЗ РК «Артериальная гипертензия»-2019 г. при хорошей переносимости антигипертензивной терапии?
130-139 мм рт.ст.
110-119 мм рт.ст.
120-129 мм рт.ст.
140-149 мм рт.ст.
150-159 мм рт.ст.
24-летняя женщина обратилась по поводу усталости. В течение последних 6 месяцев она обращалась к врачам несколько раз. Во время приема она заигрывает с врачом. Одевается очень модно и ярко. Когда в кабинет заходит медсестра женщина начинает нервничать и прекращает разговаривать. На замечание врача, что она ведет себя неуместно, пациентка начинает плакать и говорить, что «ее никто не понимает, и ей надо, чтобы ее только выслушали». Потом она говорит, что будет дальше искать врача, который ей поможет, но не надеется на успех. Какой диагноз наиболее вероятен?
Истерическое расстройство личности
Шизоидное расстройство личности
Нарциссическое расстройство личности
Зависимое расстройство личности
Пограничное расстройство личности
Вызов врача общей практики на дом. После употребления вяленой рыбы, купленной на стихийном рынке, у 25-летнего мужчины через 16 часов появились тошнота, рвота, слабость, вялость, двоение в глазах. Объективно: снижение мышечного тонуса, анизокория, вялый глоточный и сухожильный рефлексы. Ваш предварительный диагноз?
Ботулизм.
Пищевая токсикоинфекция.
Острый энцефалит.
Сальмонеллёз.
Острый менингит.
Участковый врач пришел по вызову к 2-х летнему ребенку. Ребенок посещает детский сад, болен 2-й день. Жалобы на повышение температуры тела 38 -39º, головную боль, вялость, однократную рвоту, сыпь на теле. Высыпания на коже появляются к вечеру или на следующий день с момента острого проявления заболевания. Сыпь мелкая, имеет точечную форму и яркий красный цвет. Началось высыпание с области шеи и лица образованием мелких пятнышек. Далее сыпь возникла на внутренней поверхности бедер, на туловище: груди, по бокам живота. Зона носогубного треугольника свободна от сыпи. При фарингоскопии – «пылающий зев»». Ваш предварительный диагноз?
Скарлатина
Корь
Псевдотуберкулез
Краснуха
Ветряная оспа
72-летний мужчина обратился по поводу тяжелой, острой боли в правой ноге в течение 2 часов. Симптомы появились внезапно, когда она занимался домашними делами. Боли нет в покое и при физической нагрузке. Вчера он вернулся после 6-часовой поездки на автобусе. В анамнезе – артериальная гипертензия, принимает рамиприл. Пульс 105/мин, АД 140/90 мм рт ст. Правая нижняя конечность прохладная на ощупь. Что из нижеперечисленного наиболее вероятно может подтвердить источник данного состояния этого пациента?

Эхокардиография
Допплерография
Биопсия поверхностной вены
Ангиография
Манометрия
45-летняя женщина с гипотиреозом обратилась по поводу прогрессирующей усталости, вялости, болей в эпигастральной области после еды. При осмотре – бледность конъюнктивы. В анализах сыворотки крови – снижение гемоглобина и повышение уровня гастрина. ЭФГДС – воспаление тела и дна желудка. Биопсия антрального отдела желудка – гиперплазия антральных G-клеток

Что является наиболее вероятной причиной симптомов у пациента?
разрушение париетальных клеток
гиперплазия слизистых клеток
деструкция клеток
S-клеточная гиперлазия
разрушение I-клеток
Женщина 54 лет жалуется на боль в пояснице, которая иррадиирует в правую ногу. Со слов боль появилась после подъема тяжести, стала усиливаться при движении, при длительном нахождении в одной позе. Положение на левом боку, с согнутой правой ногой в коленном и тазобедренном суставах - приносило облегчение. Объективно: спина фиксирована в слегка согнутом положении, резко ограничен наклон в больную сторону, отмечается выраженное напряжение паравертебральных мышц. Боль локализована по задней поверхности правой ноги до 5пальца и пятки. Чувствительность снижена по наружной поверхности стопы, подошвы справа. Симптом Ласега положителен справа, Ахиллов рефлекс справа отсутствует. На каком уровне позвоночного столба наиболее вероятно патология?
L5-S1
Th 12- L1
L1-L2
L3-L4
L2-L3
Женщина со сроком гестации 27-28 нед. Жалуется на повышение температуры, сухой кашель, боль в грудной клетке, одышку. В городе отмечается эпидемический подъем заболеваемости гриппа и ОРВИ. Температура 39,40С, в зеве слизистая ярко гиперемирована, задняя стенка глотки зернистая. Инъекция сосудов склер. В легких жесткое дыхание, хрипов нет. Тоны сердца приглушены, ЧСС 130 уд/мин, АД 90/60 мм.рт.ст., ЧДД – 32. В OAK: эр.-3,2х1012 /л, гем. -115 г/л, Л. -3,2x109 /л, э. – 3%, п\я – 15%, с\я – 60%, м – 7%, л – 15%, СОЭ - 15 мм/ч. Какая тактика наиболее целесообазна?
экстренная госпитализация, основной препарат осельтамавир
лечение амбулаторно, основной препарат азитромицин
лечение в условиях дневного стационара, детоксикационная терапия
плановая госпитализация в терапевтическое отделение, ацикловир
экстренная госпитализация, основной препарат цефтриаксон
46-летняя женщина обратилась к врачу по поводу болей в суставах. В течение 15 лет страдала от болей в суставах, но не обращалась за помощью. При осмотре – Т тела 37,4ᴼС, пульс 97 уд/мин, АД – 130/80 мм рт ст. При осмотре – бледность слизистой оболочки полости рта; безболезненные подкожные узелки на обоих локтях. Дистальные межфаланговые суставы обеих рук согнуты. Диапазон движения в пальцах ограничен. Печень +6 см из-под реберной дуги, селезенка пальпируется на 4 см ниже левого реберного края. ОАК – гематокрит 33%, лейкоциты – 1800/мм3, нейтрофилы 35%, лимфоциты 60%, тромбоциты 130 000/мм3. Какой маркер наиболее специфичен для этого заболевания?
Антитела к циклическому цитруллинированному пептиду
Ревматоидный фактор
ANA – антинуклеарные тела
Антитела к двухцепочечной ДНК
Антитела к Смит-антигенам
Юноша 17 лет жалуется на двоение при взгляде вправо, слабость в ногах, пошатывание при ходьбе, периодическую задержку мочеиспускания. В 15 лет - в течение 3-5 дней отмечал двоение при взгляде вправо. Месяц назад после стресса почувствовал слабость в ногах и шаткость при ходьбе. В неврологическом статусе: легкое сходящееся косоглазие справа, диплопия при взгляде вправо, скандированная речь, нижний спастический парапарез с высокими сухожильными, отсутствуют брюшные рефлексы, неустойчивость в позе Ромберга, походка с широко расставленными ногами. На глазном дне – побледнение височных половин сосков зрительных нервов. Какой из нижеперечисленных клинических диагнозов наиболее вероятный?
рассеянный склероз
боковой амиотрофический склероз
острый рассеяный энцефаломиелит
сирингомиелия
полиомиелит
Юноша 18 лет, внезапно потерял сознание на короткий отрезок времени. При осмотре: сознания- оглушение, головная боль, рвота, ригидность затылочных мышц, симптом Кернига и Брудзинского. Брадикардия, повышение температуры до 38°С. В общем анализе крови лейкоцитоз. Больной пропунктирован. Спинномозговая жидкость окрашена в красный или желтоватый цвет. Какой дополнительный метод диагностики необходимо назначить для уточнения диагноза?
КТ головного мозга
эхо-энцефалография
Электроэнцефалография
Реоэнцефалография
Коагулограмма
33-летний мужчина обратился по поводу постепенно нарастающей боли в ногах в течение 2-х месяцев. Боли возникали после длительной ходьбы, проходили в покое. На прошлой неделе боль была постоянная и сопровождалась ощущением жжения. Также были преходящие болезненные узелки по ходу вен ног в течение 4 месяцев, которые проходили самопроизвольно. Курит 2 пачки сигарет ежедневно в течение 15 лет. При осмотре – язвы на дистальной части левого большого, второго и пятого пальцев. Какое вмешательство наиболее вероятно уменьшит риск ампутации конечности у пациента?
Отказ от курения
Компрессионные чулки
Симвастатин терапия
Назначение эноксопарина
Преднизолоновая терапия
Юноша 16 лет, жалобы на высокую температуру, головную боль, головокружение с тошнотой, эмоциональную лабильность, иногда отмечаются галлюцинации. Данное состояние беспокоит в течении трех дней. Объективно: гиперкинезы в конечностях, сходящееся косоглазие, диплопия, гипергидроз, легкая пирамидная недостаточность со снижением мышечного тонуса. В анамнезе частые ангины. АСЛ-О – 220 МЕ/мл. Какой предварительный диагноз наиболее вероятный?
нейроревматизм
Нейросифилис
Нейробруцеллез
хорея Гентингтона
рассеянный склероз
55-летняя женщина с жалобами на сильные влагалищные кровотечения в течение трех дней, использует до пяти прокладок в день. Менопауза наступила 1 год назад. Менархе в 10 лет. В анамнезе – сахарный диабет и гипотиреоз, принимает метформин и левотироксин. Курила по 1 пачке сигарет в течение 20 лет, бросила курить 5 лет назад. Рост 165 см, вес 86 кг, ИМТ 32. При осмотре – нормальная шейка матки и легкая атрофия влагалища. Матка и придатки не пальпируются. Трансвагинальное УЗИ – толщина эндометрия 6 см. Биопсия эндометрия – атипичных клеток нет. Какая тактика наиболее целесообразна?
прогестиновая терапия
тотальная гистерэктомия
эстроген вагинальный крем
наблюдение
анастрозол терапия
У юноши 19 лет после гриппа появились боли в нижней челюсти справа приступообразного характера, продолжительностью 2-3 сек. Приступы возникают при разговоре, жевании, умывании. В неврологическом статусе: болезненность в точке выхода III ветви V нерва справа, курковая зона в области угла рта справа. Другой неврологической симптоматики нет. Глазное дно без патологии. МРТ головного мозга без патологии. Какой клинический диагноз наиболее вероятный?
невралгия III ветви тройничного нерва справа
острая невропатия лицевого нерва справа
невралгия I ветви тройничного нерва справа
невралгия II ветви тройничного нерва справа
острая невропатия подъязычного нерва справа
Женщина 65 лет, На фоне повышения АД до 195/110 мм рт.ст. появилось нарушение речи - везапно перестала говорить. В неврологическом статусе: сознание ясное, зрачки D=S, сглажена правая носогубная складка, моторная афазия, правосторонний гемипарез с высоким мышечным тонусом и высокими сухожильными рефлексами, с симптомом Бабинского. Какой из нижеперечисленных диагнозов является наиболее вероятным?
ишемический инсульт
острая гипертоническая энцефалопатия
транзиторная ишемическая атака
серозный менингит
геморрагический инсульт
Женщина 41 год обратилась к врачу с жалобами на сонливость, апатию, запоры, набрала в весе. Считает себя больной в течение 2х лет. Об-но: рост 150см, вес 100кг. Кожные покровы сухие, пастозные, бледность слизистых оболочек, язык утолщен с отпечатками зубов по краям. АД 110/70 мм рт.ст. ЭКГ – синусовый ритм с ЧСС-58 уд. в мин., горизонтальное положение ЭОС. Общий анализ крови: Нв 100 г/л. Холестерин общий крови – 9 ммоль/л. УЗИ щитовидной железы – уменьшение размеров железы. Какой из нижеперечисленных диагнозов наиболее вероятен?
гипотиреоз
диффузный эутиреоидный зоб
диффузный токсический зоб
подострый тиреоидит
киста щитовидной железы
Мужчина 45 лет с нормальной массой тела лечится у врача общей практики по поводу сахарного диабета 2 типа. Лечение диетой оказалось неэффективным, гликемия в течение суток от 10 до 15 ммоль/л. Сахара в моче нет. Какая тактика лечения является наиболее рациональной?
препараты сульфанилмочевины + инсулин
интенсифицированная инсулинотерапия
препараты сульфанилмочевины+ бигуаниды
10 ЕД инсулина семиленте
диета + бигуаниды
У девушки 17 лет грипп с высокой температурой. При осмотре беспокойна, тревожна, суетлива, отвлекаема. Не знает где находится, в окружающей обстановке ориентировка нарушена. Часто вскакивает с постели, что-то ищет под подушкой, заглядывает под кровать, копается в постельном белье, шарит рукой по стене, пытаясь поймать воображаемых насекомых. Временами испуганно озирается, разговаривает сама с собой. Какая тактика наиболее целесообразна?
госпитализировать в инфекционную больницу
начать лечение осельтамавиром
начать детоксикационную терапию
начать транквилизаторы
госпитализировать в психдиспансер
Мужчина 40 лет, социально благополучный, обращается с затяжными приступами болей, напоминающих почечную колику. Тянущие боли в области поясницы сохранялись в течении 4-х недель. При обследовании, включающем УЗИ и рентгенографию, патологии со стороны почек не выявлено. Назначение спазмолитиков эффекта не дало. Больной тревожится о своем будущем, плохо спит, хуже себя чувствует утром, вечером боли практически не беспокоят. Считает что врачи не хотят обращать должного внимания его заболеванию. Какой из нижеперечисленных диагнозов наиболее вероятный
Маскированная депрессия
Фобический синдром
Шизофрения
Неврастения
Нераспознанная органная патология
Женщина 35 лет, жалуется на приступы резкой головной боли с тошнотой и рвотой. Считает себя больной в течение последнего года, когда стали появляться подобные приступы, которые продолжались в течение получаса. Приступ заканчивается обычно обильной полиурией и последующей выраженной слабостью. Связывает с отрицательными эмоциями. Об-но: кожные покровы бледные, влажные. Тремор конечностей. Тоны сердца приглушены, тахикардия. АД 230/140 мм. ЧСС 110 уд/мин. Общий анализ крови: Л – 10,2х109; глюкоза крови 8,8 ммоль/л. Какой из перечисленных предварительных диагнозов наиболее вероятен?
феохромоцитома
сахарный диабет
болезнь Иценко-Кушинга
синдром Кона
гипоталамический синдром
На приеме у врача общей практики женщина 53 лет. После трагической смерти мужа, больная обвиняет себя в том, что она не смогла предотвратить гибель мужа, недостаточно была внимательна к нему, со слезами рассказывала об этом врачу, высказывала суицидальные мысли. О каком синдроме идет речь?
Депрессивный синдром
Кататонический синдром
Обсессивный синдром
Паранойяльный синдром
Дементный синдром
На приеме у врача общей практики мальчик 11 лет. Бабушка привела его на обследование в связи с тем, что он отстает от школьной программы. Мальчик легко идет на контакт, речь развита, читает, кругозор не соответствует возрасту, однако память хорошая, способность к абстрактному мышлению не ограничена. Из анамнеза: беременность и роды без особенностей, раннее развитие без отставания в психомоторном развитии. Последние годы родители выпивают, мальчик часто пропускает школу. О каком состоянии можно думать?
Педагогическая запущенность
Шизофрения
Расстройство личности
Психический инфантилизм
Деменция
У 26-летнего мужчина возникла сильная загрудинная боль. В анамнезе отрицает прием наркотиков и алкоголя, не курит. На ЭКГ – подъем сегмента ST выше изолинии в 5 грудных отведениях. Употребление какого вещества, наиболее вероятно, будет причиной?
Стимуляторы
Фенциклидин
Седативные средства
Галлюциногены
Алкоголь
Женщина, 47 лет, жалуется увеличение массы тела, сухость кожи, затруднение речи. Объективно: рост 164 см, вес 98 кг. Выраженный гиперкератоз. Лицо пастозное, язык с отпечатками зубов по краям. Тоны сердца глухие, брадикардия. АД 130/80 мм рт. ст. В крови: эритроциты 3,5х1012/л, гемоглобин 89 г/л, лейкоциты 4,5х109/л, СОЭ 13 мм/час. Сахар крови 3,5 ммоль/л. Йод, связанный с белками сыворотки 300 ммоль/л. Какую группу лекарственных средств НАИБОЛЕЕ целесообразно назначить?
Тиреоидные препараты
Препараты йода
Мочегонные средства
Глюкокортикостероиды
Противовоспалительные средства
Ранее здоровый 28-летний мужчина обратился по поводу головокружения и сердцебиения в течение 2 дней. До появления симптомов он посетил вечеринку, где выпил много спиртного. На ЭКГ – нерегулярный ритм с узкими QRS-комплексами и без видимых р-волн. Какой диагноз наиболее вероятен?

Мерцательная аритмия
Синдром слабости синусового узла
Желудочковая фибрилляция
Гипертрофическая обструктивная кардиомиопатия
Дилатационная кардиомиопатия
Мужчина 50 лет, обратился с жалобами на сильную одышку, усиливающуюся при минимальной физической нагрузке. В анамнезе перенесенный туберкулез легких. Все началось 1,5 часа назад на работе. При подъеме тяжести появилась внезапная боль в левой половине грудной клетки и кашель. При перкуссии- тимпанит над всей поверхностью правой половины грудной клетки. Дыхание справа не прослушивается. На рентгенограмме грудной клетки - смещение сердца влево. Пульс 122 уд/мин. Какой диагноз?
Спонтанный пневмоторакс
Тромбэмболия легочной артерии
Ателектаз правого легкого
Ателектаз левого легкого
Колапс левого легкого
У вас на приеме ребенок 3 дней жизни. Мама жалуется на плач ребенка при пеленании. При пальпации правой ключицы ребенок начинает беспокоится, при движения в правом плечевом суставе. Какая НАИБОЛЕЕ подходящая тактика лечения?
мягкая повязка Дезо
гипсовая иммобилизация в физиологическом положении
гипсовая повязка Дезо
иммобилизации не требуется
восьмиобразная повязка.
Женщина 42 лет. Получает антигипертензивную терапию. Рассчитанный 10-летний показатель SCORE <1%. В анализах: холестерин 6,87 ммоль/л, ЛПНП 3,93 ммоль/л, ЛПВП 0,88 ммоль/л, триглицериды 2,24 ммоль/л, коэффициент атерогенности 5,09. Врач назначил симвастатин. До какого целевого уровня необходимо снижение уровня ЛПНП (УД – IIa C) согласно протоколу МЗ РК «Артериальная гипертензия» от 03.10.2019 г.?
<3,0 ммоль/л
<3,2 ммоль/л
<3,4 ммоль/л
<3,6 ммоль/л
<3,8 ммоль/л
Два года назад больной перенес холецистэктомию по поводу калькулезного холецистита. Через 6 месяцев появились боли в правом подреберье, темная моча. В биохимическом анализе крови – общий билирубин 120 мкмоль/л, прямой 87 мкмоль/л. Какой из перечисленных методов диагностики является в данном случае НАИБОЛЕЕ информативным?
эндоскопическая ретроградная панкреатохолангиография
УЗИ
сцинтиграфия печени
внутривенная холангиография
лапароскопия.
Женщина 65 лет, обратилась жалобами на сухость во рту, постоянную жажду и общую слабость с. Из анамнеза: 2 года назад она оперирована по поводу острого панкреатита. При осмотре живот мягкий, безболезненный. На УЗИ брюшной полости размеры поджелудочной железы в пределах нормы, без патологических образований. Лабораторное исследование, позволяющее уточнить развития отдаленного осложнения острого панкреатита?
анализ крови на глюкозу
анализ мочи на диастазу
анализ крови на диастазу
анализ кала на скрытую кровь
анализ крови на свертываемость
У пациента, 45 лет, после физической нагрузки появились схваткообразные боли по всему животу. С течением времени появились тошнота, рвота и вздутие живота, перестали отходить газы. Данный симптомокомплекс характерен для?
острая кишечная непроходимость
прободная язва желудка
острый панкреатит
острый аппендицит
острый холецистит
68-летний мужчина обратился по поводу усталости и мышечных спазмов в течение последних 4 недель. Отмечает эпизоды покалывания в обеих руках. Хронический кашель в течение 10 лет, в анамнезе – хронический бронхит, АГ и остеортроз коленных суставов. Принимает сальбутамол, ибупрофен, рамиприл. Отец умер от рака легких. Курит 1 пачку сигарет в день в течение 45 лет. ИМТ 22 кг/м2. Пульс 60/мин, АД 115/75 мм рт ст. При измерении АД – спазм запястья. Анализы – гематокрит 41%, лейкоциты – 5800/мм3, тромбоциты – 195 000/мм3; активность щелочной фосфатазы 55 ед/л. ЭКГ – синусовый ритм, с удлиненным интервалом QT. Что является причиной данного состояния?
Побочный эффект лекарств
Множественная эндокринная неоплазия
Эктопическая выработка гормонов
Разрушение околощитовидных желез
Недостаток витамина D
Девочка 4 месяцев. Родилась недоношенной, с весом 2100 грамм, ростом - 44см. Растет и развивается соответственно возрасту. На диспансерном учете у невропатолога с диагнозом: «Перинатальная энцефалопатия». Прививки получает в срок. Какая наиболее вероятная вакцинация в этом возрасте?
АбКДС3 + ИПВ3+ Hib3+ВГВ
АбКДС3 + ИПВ3+ Hib3
ККП + ИПВ4+пневмо
RV АбКДС + RV Hib
RV БЦЖ + RV АДС+ RV ККП
Женщина 37 лет, обратилась к врачу с жалобами на боль и припухлость лучезапястного сустава, сильную болезненность и слабость в руке, невозможность сжать кисть и удержать в ней предмет; субфебрилитет по вечерам, ноющие боли в коленных суставах. Хирург ввел ей дипроспан внутрисуставно, что дало почти моментальное облегчение. На следующий день у женщины сустав сильно опух, покраснел, появилась дергающая боль, поднялась температура. Какой диагноз наиболее вероятен?
острый гнойный артрит, вызванный стафилококком
псевдосептический артрит при ревматоидном артрите
острый подагрический артрит
острый постстрептококковый артрит
бруцеллезный артрит, спровоцированный введением дипроспана
48-летний мужчина обратился в приемное отделение по поводу внезапного появления в течение часа боли в груди и одышку. Боль сильная, иррадирует в левую руку и левую часть спины. В анамнезе артериальная гипертензия, принимает гидрохлортиазид и лизиноприл. Курит 1 пачку сигарет ежедневно в течение 30 лет. Состояние тяжелое, PS 105/мин, ЧДД 22 мин, АД 170/90 мм рт ст. На ЭКГ – синусовая тахикардия и гипертрофия левого желудочка. Какая тактика наиболее целесообразна?
Назначить бета-блокатор
Начать терапию гепарином
Хирургическое вмешательство
Назначить аспирин
Наблюдение
Мальчик 14 лет. Жалобы на слабость, головокружение, снижение аппетита. Стал плохо переносить обычные спортивные нагрузки. Слабость и утомляемость объясняет нежеланием есть мясо. При осмотре бледность кожи и слизистых оболочек, ладоней. Язык бледный, сглажены сосочки. Дыхание везикулярное. Сердечные тоны приглушены, систолический шум на верхушке, ЧСС 92 в мин. АД 100/60 мм.рт.ст. Живот мягкий, болезненный в эпигастральной области, печень пальпируется на 1 см из-под реберной дуги. ОАК: эритроциты 3,58х1012/л, HB 89 г/л, Ht 30 %, СОЭ 28 мм/час. Какое обследование с наибольшей достоверностью позволит выявить причину данного состояния:
ЭФГДС с биопсией
дыхательный тест
УЗИ органов брюшной полости
Общий белок, АЛТ, АСТ, СРБ
стернальная пункция
Девочка 14 лет жалуется на боли в эпигастрии, ближе к левой половине, иррадиирующей в спину слева. Боли возникают после еды, особенно обильной, жирной и острой. Также боль провоцируется употреблением гамбургеров. Симптомы наблюдаются в течение 6 мес. Из-за избыточного веса девочка периодически устраивает разгрузочные дни. При полном голодании, приеме только воды боли исчезают. Стул неустойчивый, чаще разжижженый. Наиболее информативный показатель для подтверждения диагноза:
уровень эластазы-1
УЗИ органов брюшной полости
МРТ органов брюшной полости
содержание амилазы, холестерина, триглицеридов
уровень гликированного гемоглобина
Девочка 9 мес. Жалобы на повышение температуры тела до 38,7ОС, отказ от груди, рвота после кормления. Антенатальный и постнатальный анамнез без особенностей. Прививки в срок, без реакций. При осмотре катаральных явлений со стороны верхних дыхательных путей нет. Ригидность затылочных мышц. Кожные покровы бледные с сероватым колоритом, губы суховаты. В легких жестковатое дыхание, хрипов нет, ЧД 52 в мин. Сердечные тоны звучные, ритмичные, ЧСС 142 в мин. Живот мягкий, болезненный при пальпации. Мочится мало. Дизурия. Стул жидкий. Наиболее вероятный диагноз:
инфекция мочевых путей
внебольничная пневмония
острая кишечная инфекция
острый гнойный отит
вирусный менингит
Мать ребенка 5 лет обратилась к врачу по поводу частого сухого кашля, беспокоящего ребенка ночью и рано утром в течение трех месяцев. В последнее время после физической нагрузки (бег, прыжки) периодически возникает свистящее дыхание, которое самостоятельно проходит после отдыха. Что может явиться причиной данного состояния:
бронхиальная астма
ВПС с гиперволемией в малом круге кровообращения
острый неревматический кардит, НК 2 А.
хронический бронхит
рецидивирующий обструктивный бронхит
2-хлетнего мальчика беспокоят усталость и пожелтение кожи в течение 2-х дней. Неделю назад был насморк после переохлаждения. Прошел пятидневный курс по поводу желтухи новорожденных. Жизненные показатели в пределах нормы. Объективно – желтушности кожи и конъюнктивы. Селезенка пальпируется на 3 см ниже реберной дуги. Hb 9.8 г/дл, средняя концентрация гемоглобина в эритроците (МСНС) 38%. Тест Кумбса отрицательный. В мазке:

Развитие какого осложнения наиболее вероятно?
Холецистит
Острый миелогенный лейкоз
Синдром Пламмера-Винсона
Остеомиелит
скелетные деформации
32-летняя женщина обратилась к врачу из-за усталости, болезненности груди, увеличение частоты мочеиспускания и периодическую тошноту в течение 2-х недель. Последний менструальный цикл был 7 недель назад. В анамнезе эпилепсия, которую лечили карбамазепином. При осмотре – патологии не выявлено. Тест на беременность положительный. Наибольший риск какого осложнения наиболее вероятен у будущего ребенка?
дефект нервной трубки (менингоцеле)
почечная дисплазия
изменение цвета зубов
нейросенсорная тугоухость
порок развития конечностей
Ребенок 3 месяцев в тяжелом состоянии. Болен 4 день. Одышка, пероральный цианоз в покое Частота дыхания 84 в мин, дыхание шумное, слышно на расстоянии. Умеренные проявления токсикоза с эксикозом. Над легочными полями перкуторно - тимпанит, аускультативно: удлиненный затрудненный выдох, по всем полям влажные мелкопузырчатые хрипы. SO2 89 %. ОАК RBC 4,3х1012/л, HB 112 г/л, PLT 210х109/л, 1 WBC 12,1х10х109/л с/ядерные 52%, эозинофилы 2%, моноциты 7%, лимфоциты 38% СОЭ 20 мм/час Какой из перечисленных ниже диагнозов наиболее вероятный:
острый бронхиолит
острый бронхит
острый обструктивный бронхит
двусторонняя очаговая пневмония
интерстициальная пневмония
Участковый врач осматривает ребенка 1 года по вызову на дому. Жалобы матери на повышение температуры тела до 38,80С, насморк, кашель в течение 4 дней. При осмотре, одышка, периоральный цианоз, адинамия, сонливость, грудь сосет неохотно. В легких выслушиваются мелкопузырчатые хрипы, больше справа. ЧД 62 в мин. Сердечные тоны приглушены, ЧСС 138 в мин. . SO2 92 %. Что из перечисленного требует госпитализации в стационар:
вялость, сонливость, снижение аппетита
тахипноэ, тахикардия
повышение температуры, нарушение сна
влажные мелкопузырчатые хрипы в легких
показатель сатурации кислорода
Ребенок 1 год. Жалобы на одышку, кашель, нарушение аппетита, вялость, сонливость. Температура - 39,1оС. На рентгенограмме – очагово-сливная инфильтрация в правой нижнемедиальной зоне. После назначенного лечения цефтриаксоном на 3-е сутки температура в вечерние часы 37,6оС. Сон восстановился, аппетит лучше, стал активнее днем. Ваша дальнейшая тактика?
продолжить тот же антибиотик до нормализации температуры
перевести ребенка на альтернирующий режим введения антибиотика
заменить антибиотик на цефалоспорины 4 поколения
сделать бак.посев мокроты и определить чувствительность возбудителя
продолжить антибиотик до полного
Мама 6-дневного новорожденного обратилась по поводу пожелтения за один день его кожи и глаз. Мать сообщает, что кормит сына около 7 раз в день. Отмечает, что вчера у сына было 2 мокрых подгузника и два испражнения. Родился на 38 нед беременности и весил 3500 г; сейчас весит 3000г. При осомтре – Т тела 37ᴼС, пульс 180/мин, АД 75/45 мм рт ст., желтушность склер, распространенная желтуха, сухие слизистые оболочки. В крови – общий билирубин 9 мг/дл, непрямой 0,7 мг/дл, АСТ 15 Ед/л. Что из нижеследующего является наиболее подходящим для следующего шага в ведении этого пациента.
Увеличение частоты грудного вскармливания
Введение внутривенно иммуноглобулина
Назначение фенобарбитала
УЗИ брюшной полости
Светолечение
9-летний мальчик обратился по поводу болей при глотании и жжения во рту в течение последних 10 дней. За последние 3 недели появилась быстрая утомляемость. У отца были камни в желчном пузыре, холецистэктомия в 30 лет. Объективно - Т тела 37,7ᴼС, пульс 105/мин, АД 110/65 мм рт ст., ЧДД 28/мин. Бледность слизистых оболочек, умеренная желтушность склер, опухший красный язык. В крови – Hb 9,8 г/дл, MCV 102 мкм3, ретикулоциты 0,4%; эритроциты без центральной бледности. Что могло бы предотвратить жалобы пациента?
Назначение фолиевой кислоты
Холецистэктомия
Безглютеновая диета
Вакцинация против пневмококков
Инъекции витамина В12
Мальчик 12 лет жалуется на боли в крупных суставах, припухлость в левом локтевом суставе, мышечную слабость, лихорадку до 39,8º с потрясающими ознобами. Температура повышается в утренние часы и после обеда, держится около 2-х часов, далее снижается с профузным потоотделением до нормы. Лечили антибактериальными препаратами без эффекта. С момента начала заболевания сильно похудел. При осмотре выявлены полиморфная сыпь на туловище и конечностях, увеличение подчелюстных и подмышечных лимфоузлов, и гепатоспленломегалия. ОАК: Hb – 90 г/л, лейкоциты – 20,6х109/л, тромбоциты – 230х109 /л, СОЭ – 55 мм/час. СРБ – 50 мг/мл, АНА – отрицательно, РФ – 5 Ме/мл, АСЛ-О – 130 МЕ/л, ферритин – 1000 мкг/л, прокальцитонин – 0,3 нг/мл. С какой патологией необходимо дифференцировать в первую очередь?
Лейкозы
системный васкулит
системная красная волчанка
острая ревматическая лихорадка
сепсис
На приеме у врача ребенок 9 мес. На грудном вскармливании. Объективно: снижен мышечный тонус, не ползает, живот увеличен, при сидении выражен кифоз грудного отдела позвоночника. Со стороны костной системы макроцефалия, большой родничок 2х2 см, уплощение и облысение затылочной кости, лобные и теменные бугры, рахитические четки, гаррисонова борозда Масса тела 9200 г. Задержки физического и психомоторного развития нет. Врач назначила лечение. Укажите дозу холекальцеферола (вит Д3) для лечения гиповитаминоза у данного ребенка:
2000-2500 МЕ/сутки в течение 1 мес
500 МЕ/сутки в течение 1 года
1000-1500 МЕ/сутки в течение 1 мес
1500-2000 МЕ/сутки в течение 1 мес
3000-3500 МЕ/сутки в течение 1 мес


14-летняя девочка с жалобами на повторяющиеся носовые кровотечения в течение нескольких месяцев. Эпизоды возникают неожиданно и прекращаются через несколько минут, если наклоняется и наклоняет голову вперед. Менструации регулярные, обильные, интервал 27 дней. Последний менструальный цикл был три недели назад. В крови – Hb 11 г/дл, гематокрит 34%, лейкоциты 7000/мм3, тромбоциты 180 000/мм3, протромбиновое время 13 сек, частичное тромбопластиновое время 45 сек, время кровотечения 10 мин.
Какая тактика наиболее целесообразна?
Назначение десмопрессина
Назначение витамина К
Назначение фолиевой кислоты
Назначение иммуноглобуллинов
Назначение преднизолона
Мальчик 5 лет, жалуется на высыпание на лице и по всему телу, лихорадку до 39,5º, боли и припухлость в локтевом суставе справа. Из анамнеза: со слов мамы вчера появилась высыпание на лице и к вечеру распространились по всему телу. Ребенок посещает детский садик больше 1 месяца, вакцинации после 6 месяца жизни не получал. Объективно: розово-красная пятнисто-папулезная сыпь по всему телу, артрит правого локтевого сустава. Заднешейные и затылочные лимфоузлы пальпируются. Анализ крови: Hb - 121 г/л, лейкоциты - 10х109 /л, СОЭ - 20 мм/час, СРБ– 8 мг/мл, АСЛ-О - 1:100 МЕ/л, РФ – 5 МЕ/мл. Какая наиболее вероятный этиологический фактор возникновения заболевания?

вирус краснухи
вирус кори
вирус Коксаки
вирус ветряной оспы
возбудитель скарлатины
Мальчик 10 лет жалуется на боли и припухлость левого коленного сустава с выраженным ограничением движения, появления синяков на коже конечностей. Мама заметила повышенную кровоточивость десен, носовые кровотечение за последний месяц. Из анамнеза: отец и тетя мамы страдали повышенной кровоточивостью. При осмотре: кровоизлияния в подкожную область по типу «синяков», левый коленный сустав увеличен в объеме, кожа над ним гиперемирована и горячая на ощупь. ОАК: Hb – 100г/л, лейкоциты – 4,6х109/л, тромбоциты – 177х109 /л, СОЭ – 25 мм/час; ОАМ – гематурия; Коагулограмма: АЧТВ – 60 сек, ТВ – 15 сек, фибриноген – 3 г/л, ристоцетин-кофакторная активность – в норме; РФ – 7 МЕ/мл, СРБ – 22 мг/мл. Рентген коленных суставов (фото) Какой метод обследование целесообразно провести для верификации диагноза?

определения факторов свертывания крови VII и IX
маркеры гепатитов В и С
кровь на ВИЧ
определение фактора Виллебранда
определения HLA-B27
Новорожденный 4 дня, исключительно на грудном вскармливании, при рождении весил 4 000 г. Мама обратилась по поводу пожелтения его глаз и кожи. Моча и стул нормальные. Во время беременности у мамы был гестационный сахарный диабет. У его старшего брата была желтуха в период новорожденности. Жизненные показатели в пределах нормы. Печень пальпируется на 1 см ниже правого реберного края. В крови - Hb 17 г/дл, ретикулоциты 0.5%, общий билирубин 21.2 мг/дл, прямой билирубин 2 мг/дл, непрямой билирубин 19.1 мг/дл, тест Кумбса отрицательный. Что является наиболее целесообразным в управлении заболеванием?
Светолечение
Назначить иммуноглобулин
Увеличение частоты кормления грудью
Наблюдение
Заменить грудное вскармливание молочными смесями
У 4-х летней девочки нарастает желтуха, которая началась 8 дней назад. У нее были подобные эпизоды в прошлом. Ее отец перенес спленэктомию в подростковом периоде. При осмотре легкая спленомегалия. В крови: гемоглобин 10,1 г/дл, MCHC 41% Hb/клетке, MCV 81 мкм3, тромбоциты – 250 000/мм3, RDW 16%, ретикулоциты 11%, СОЭ 10 мм/час. В мазке крови – сфероциты. Какой тест наиболее специфичен для подтверждения состояния пациентки?
Тест Кумбса
Тест на связывание эозин-5-малеимида
электрофорез гемоглобина
Уровень ферритина в сыворотке
Тест осмотической хрупкости
Мальчик 6 месяцев, жалобы на сонливость, потерю аппетита, отказ от питья, повышение температуры до 39,8. Из анамнеза: болен в течении 3-х дней. Объективно: дыхание затрудненное, частота дыхания за 1 минуту 68. Отмечается втяжение грудной клетки. Правильная тактика врача по программе ИВБДВ
антибиотик в/м, срочная госпитализация
антибиотик в/м, лечение на дому
антибиотик в/м, сальбутамол
антибиотик в/м, плановая госпитализация
сальбутамол, антибиотик через рот
В родильном доме родился мальчик с массой тела 2100,0 гр. На 2-й день жизни появилась желтушность кожных покровов, которая стала нарастать. Объективно: гепатомегалия, петехиальная сыпь на коже туловища. В анализах: в мазке из слизистой щек обнаружены гигантские многоядерные клетки с внутриклеточными включениями в ядрах и цитоплазме. НАИБОЛЕЕ вероятный предварительный диагноз
ЦМВ инфекция
Листериоз
Токсоплазмоз
Сифилис
Иерсиниоз
Мальчик 12 лет, 2 года назад после ангины появились гиперкинезы, непроизвольные движения конечности и тремор кистей, артрит левого коленнного сустава, коротким курсом принимал преднизолон 20 мг/сутки и антибиотики с положительным эффектом. В настоящий момент проходит диспансерный осмотр, жалоб не предъявляет. При физикальном исследовании выявляется систолическим шум на 2 и 5 точке выслушивания и там же 2 тон акцентирован, ослабления Iтона на верхушке. Печень не увеличена, отеков нет. Анализ крови: Hb - 120 г/л, лейкоциты - 5,0х109/л, СОЭ - 10 мм/час, С-реактивный белок – 3,4 мг/мл, АСЛ-О - 1:100МЕ/мл. Какая продолжительность вторичной профилактики бензатин бензилпенициллином наиболее целесообразно у пациента?
До достижения 21-летнего возраста
До 3 лет
До 10 лет после атаки
До достижения 25-летнего возраста
До достижения 40-летнего возраста
9-летний мальчик с жалобами на болезненную припухлость в правом колене, которая появилась после столкновения во время футбольного матча с другим игроком. Несколько месяцев назад у него было сильное кровотечение после удаления зуба. При осмотре выраженная болезненность и припухлость правого коленного сустава. На нижних конечностях синяки разной стадии заживления. Показатели - тромбоциты – 235 000/мм3, АЧТВ (активированное частичное тромбопластиновое время) 78 сек, протромбиновое время 14 сек, время кровотечения 4 мин. Концентрация какого показателя вероятнее всего будет снижена в плазме крови?
Тромбин
С-реактивный белок
Плазмин
Фактор VII
Фактор Виллебранда
Мама 4-месячного мальчика обратилась по поводу поражения у него на правом бедре. Вчера ему были проведены все запланированные прививки. Жизненные показатели в пределах нормы. При осмотре – на переднелатеральной поверхности правого бедра язва размером 2,0 см с уплотнением. Что является вероятной причиной данного состояния?
Активация дермальных тучных клеток
Иммунный комплекс отложения
Интрадермальный акантолиз
Инфекционное кожное воспаление
Опосредованная Т-лимфоцитамм гиперчувствительность
Женщина 48 лет, обратилась с жалобами на приступы сердцебиения, сопровождающееся ощущением нехватки воздуха, общей слабостью; потливость, снижение веса, тремор рук, раздражительность. При обследовании в момент приступа на ЭКГ (фото). Какое обследование наиболее целесообразно?
ТТГ, Т3 и Т4
Эхокардиография
Холтеровское мониторирование ЭКГ
ВЭМ-тест
уровень антител к ТПО и ТГ
Женщина 33 года, обратилась с жалобами на массивные отеки. Со слов больна около недели, после перенесенного ОРЗ, появились отеки с нарастанием, урежением диуреза. Также беспокоят высыпания по типу крапивницы, боли в суставах. Anti-HCV IgM, IgG – отрицательные, ПЦР HCV РНК – положительный. РФ – положительный, АНА - отрицательные. В моче: суточная протеинурия 5 г в сутки, в крови: общий белок 52 г/л, альбумин – 23 г/л, холестерин 7,5ммоль/л. Какой НАИБОЛЕЕ вероятный предварительный диагноз?
Хронический гломерулонефрит, ассоциированный с HCV
Острый гломерулонефрит
Геморрагический васкулит с поражением почек
Системная красная волчанка, волчаночный нефрит
IgA-нефропатия
Мать привела на прием 3-летнюю девочку из-за внезапного появления сыпи. Мать говорит, что сыпь появилась через час после того, как она купала ребенка в теплой воде. Две недели назад девочке был поставлен диагноз инфекции кожи, пролечили пенициллином. Девочка не всегда посещает запланированные приемы у врача. Девочка живет вдвоем с матерью, которая недавно потеряла работу. У матери – серьезное депрессивное расстройство, у бабушки – СКВ. Девочка избегает смотреть на врача. При осмотре – резко очерченная эритема на нижних конечностях, похожая на ожог. Что покажет дальнейшая оценка девочки?
Множественные травмы на разных этапах заживления
Язвы слизистой оболочки рта
Положительный знак Никольского
Дерматографизм
Скуловая сыпь
.Ранее здоровый 4-летний мальчик был доставлен к врачу его родителями в связи с лихорадкой, диффузной болью в суставах и сыпи на животе в течение прошлой недели. Он эмигрировал из Китая со своей семье 2 года назад. Посещает детский сад. О его прививках ничего не известно. Температура тела 38,5ᴼС, пульс 125мин, АД 100/60 мм рт ст. при осмотре – на теле полиморфная сыпь, конъюнктивит. Язык блестящий и красный, губы потрескавшиеся, руки и ноги красные и опухшие; правосторонняя передняя шейная лимфаденопатия.
INCLUDEPICTURE "https://media-us.amboss.com/media/thumbs/big_5773911f74e73.jpg" \* MERGEFORMATINET
Какой следующий шаг рекомендован для управления заболеванием?
Эхокардиография
измерение титра антистрептолизина О
ANA измерение
Анализ вирусных иммуноглобулиновых антител
Monospot тест
Мужчина, ветеринарный врач 50 лет, обраитился с жалобами на отсутствие аппетита, слабость, зуд кожных покровов, одышку, увеличение живота в течение последнего месяца, боли в ногах. В анамнезе: употреблял еженедельно 160 мл этанола. Последние три года периодически отмечалась умеренная желтуха, сопровождающаяся кожным зудом и темным окрашиванием мочи, проходящая самостоятельно. При осмотре пониженного питания, иктеричность кожных покровов со следами расчесов, тургор снижен, на груди и спине сосудистые звездочки, тремор рук, контрактура Дюпюитрена. Живот увеличен в объеме за счет асцита, пупочная грыжа из-за чего печень и селезенку пальпировать не удается. Какой диагноз наиболее вероятен?
Цирроз печени алкогольной этиологии.
Криптогенный цирроз печени.
Первичный билиарный цирроз печени.
Цирроз печени вирусной этиологии.
Гепатоцеллюлярная карцинома
12-летняя девочка обратилась по поводу сильной боли в ушах и желтых выделений из левого уха в течение последних 2 дней, зуд в этом же ухе. Занимается плаванием в бассейне. T тела 37°С, PS 76/мин, АД 110/75 мм рт ст. При осмотре – красный и отечный слуховой канал. При исследовании камертоном (камертон посередине лба) – сообщает, что слышит звук сильнее в левом ухе. Что является наиболее вероятной причиной данного состояния?
pseudomonas aeruginosa
Aspergillus
Плеоморфная замена нормальной кости
Streptococcus pneumoniae
Вирус ветряной оспы
Женщина 56 лет. Жалобы дискомфорт в эпигастрии, чувство переполнения после еды, отрыжку воздухом, иногда тухлым. В анамнезе - аутоиммунный тиреоидит. При ФЭГДС: (фото). Ее также беспокоит слабость, жжение языка, парестезии. При осмотре кожа бледная с желтушным оттенком. Какая картина крови будет выявлена наиболее вероятно?
гиперхромная анемия
Лейкопения
гипохромная анемия
Лейкоцитоз
нормохромная анемия
.Мужчина, 46 лет, жалуется на тупые боли в правом подреберье, горечь во рту, тошноту, горечь во рту, возникающие после погрешностей в диете в течение последних 3 лет. Периодически – запоры. При обследовании: ОАК: гемоглобин – 138 г/л, лейкоциты – 6,5х109/л, СОЭ - 18 мм/ч. БАК: общий билирубин - 14 ммоль/л, АСТ - 32 ЕД/л, АЛТ - 28 ЕД/л, амилаза - 87 ЕД/л, ЩФ - 56 ЕД/л. УЗИ ОБП: Общий желчный проток не визуализируется. Желчный пузырь: тонус его понижен, размеры несколько увеличены, стенка желчного пузыря утолщена (6 мм), конкрементов нет. Какая лечебная тактика?
урсодезоксихолевая кислота
креон
дротаверин
лансопразол
ЛИВ-52
36-летний мужчина обратился в травмпункт по поводу ожогов на верхних конечностях. Начата анальгетическая терапия опиодным препаратом. Вскоре у пациента появляется озноб, потливость, тошнота и боль в животе. При опросе мужчина сообщает, что он курил опиум дома, для релаксации. Какой препарат необходимо было назначить этому пациенту, чтобы не вызвать абстиненцию?
Морфий
Фентанил
Метадон
Буторфанол
Оксикодон
38-летний мужчина обратился к врачу из-за болезненной сыпи на левой ноге. Последние 2 года у него периодически появлялись эпизоды изменения цвета пальцев на холоде – от белого до синего и красного, которые потом проходили. Курит 2 пачки сигарет ежедневно в течение 20 лет. АД 115/78 мм рт ст. При осмотре – множественные ттемно-фиолетовые узелки на боковой поверхности левой стопы с окружающей эритемой. Сухие язвы на кончике правого указательного пальца. Какой диагноз наиболее вероятен?
Облитерирующий тромбофлебит
Дислипидемия
Артериит Такаясу
Полиневропатия
Тромбоз глубокиз вен нижних конечностей
Мужчина 56 лет, предъявляет жалобы на частые головные боли при перемене погоды, к концу рабочего дня. 7 лет назад обнаружена артериальная гипертензия, лечился не регулярно. 2 года назад перенес ишемический инсульт, курит 20 сигарет в день. При осмотре тоны сердца ясные, акцент 2 тона на аорте. АД 165/100 мм.рт.ст. Левая граница сердца смещена влево на 1 см. На ЭхоКГ: индекс массы миокарда левого желудочка 140 г/м2, УЗДГ сонной артерии: комплекс интима-медиа-1,1 мм, в области бифуркации сонной артерии -1,5 мм. Лечебная тактика?
фозиноприл и карведилол
монотерапия лизиноприлом
монотерапия бисопрололом
лизиноприл и валсартан
амлодипин и каптоприл
Мужчина 62 лет, отмечает повышения АД 175-190/95-110 мм рт.ст. Лечится не регулярно. Вес-100 кг, рост- 165см., объем талии-105см. Объективно: левая граница сердца на 1 см влево от левой средне ключичной линии, по 5 межреберью. На ЭКГ: ритм синусовый, индекс Соколова-Лайона – 40мм. МАУ – 300 мг/сутки. Комплекс интима-медиа сонной артерии - 1мм. Какая вероятность развития кардио-васкулярных катастроф в ближайшие 10 лет?
выше 30%
10-15%
16-20%
21-30%
до 10%
.Девушка 25 лет, жалобы на выраженную слабость, одышку при малейшей физической нагрузке, пастозность лодыжек, сердцебиение. 2 недели назад перенесла ОРВИ. Объективно: бледность, пастозность лодыжек. При аускультации: в легких – крепитация в нижних отделах, тоны сердца приглушены, систолический шум на верхушке, не проводится, ЧСС 110 мин, АД 100/70 мм рт.ст. ЭКГ (фото). В анализе крови – СОЭ – 25 мм/ч. Какой диагноз?

острый миокардит, НК2А
дилятационная кардиомиопатия, НК2А
ОРЛ, ревмокардит, НК 1
волчаночный кардит
хронический миокардит, обострение
Мужчина 68 лет. В последние 2 года отмечает появление перебоев в работе сердца, учащенное сердцебиение. Артериальная гипертензия 2 степени и ИБС в течение 15 лет. При аускультации сердца: ритм неправильный, периодически «пушечный тон», число сердечных сокращении 100 в мин, пульс-86 в мин., нерегулярный. Что Вы, ожидаете увидеть на ЭКГ больного?
зубцы P отсутствуют, расстояния R-R разные, волны f во II, III. аvF
наличие внеочередных QRS и компенсаторных пауз
периодическое выпадение QRS после зубца Р
расстояния R-R одинаковые, зубец Р перед узким QRS
расстояния R-R одинаковые, зубец Р перед QRS, Т отрицательный
42-летняя женщина обратилась в приемное отделение в связи с болью в правой верхней части живота и тошноты, беспокоящих ее в течение 2 дней. Рост 163 см, вес 91 кг, ИМТ 34кг/м2. Т 38,5°С. При осмотре – живот вздут, болезненность в правом верхнем квадранте с нормальными звуками кишечника. В анализах – лейкоцитов 14 000/мм3, общий билирубин 1,1 мг/дл, АСТ 32 Ед/л, щелочная фосфотаза 68 Ед/л. Что является наиболее вероятной причиной данного состояния?
Обструкция пузырного протока
Гепатотропный вирус
Фиброз общего желчного протока
Гепатома желчного пузыря
Фистула между желчным протоком и тонкой кишкой
21-летний мужчина доставлен скорой помощью с переломом ноги. Проведена операция с применением ингаляционного анестетика. Во время анестезии минутная вентиляция дыхательных путей заметно снижается. О каком из нижеперечисленных дополнительных эффектов в ответ на использование препарата должен был предупредить врач?
Повышенной внутричерепное давление
Увеличение скорости клубочковой фильтрации
Увеличение центрального метаболизма
Снижение порога судорог
Повышенный тонус скелетных мышц
48-летнего мужчину привела на прием жена в связи с тем, что он стал беспокойным и тревожным. Жена говорит, что он казался раздражительным в течение последних 4 месяцев. Он потерял работу, так как постоянно отпрашивался и проводил большую часть времени в телефоне, делая ставки на футбольные матчи. Он избегает всех семейных праздников, так как занимал деньги у родственников. Ранее курил 8-10 сигарет в день, но бросил 1 месяц тому назад. Пьет пиво по выходным. Осмотр - слегка дрожит, отказывается разговаривать с врачом. Ориентирован на человека, в пространстве и времени. Неврологическое обследование в норме. Какой диагноз наиболее вероятен?
Игромания
Никотиновая абстиненция
Депрессивное расстройство
Биполярное расстройство
Стрессовое расстройство
Мужчину 42 лет в течение 2 лет беспокоят боли за грудиной сжимающего характера, иррадиирующие в левое плечо и лопатку, возникающие при быстрой ходьбе и купирующиеся в покое. Ухудшение самочувствия связывает с ночными сменами и переутомлением. Об-но: АД 140/80 мм рт.ст. Тоны сердца ясные, ритм правильный, ЧСС-72 уд/мин. Велоэргометрическая проба показала высокую толерантность к физической нагрузке. Какой из перечисленных диагнозов НАИБОЛЕЕ вероятен?
Стабильная стенокардия напряжения ФК I
Стабильная стенокардия напряжения ФК II
Стабильная стенокардия напряжения ФК III
Впервые возникшая стенокардия
Прогрессирующая стенокардия
24-летняя женщина обратилась к врачу из-за беспокоящих в течение 1 года симптомов - эпизодической одышки, стеснение в груди, головокружение, потные руки, чувство обреченности. Симптомы появляются, когда она идет на прогулку или ждет в очереди в кафе. Не может покинуть дом сама в связи с тем, что боится оставаться одна, когда появляются вышеуказанные симптомы. Выходит из дома только в сопровождении друга. Вредных привычек нет. Какой диагноз является наиболее вероятен?
Агорафобия
Социальное тревожное расстройство
Паническое расстройство
Соматическое расстройство
Расстройство личности
Родственники женщины 75 лет обеспокоены ее постепенно нарастающим изменением настроения, вне зависимости от ситуации. Отмечают резкие переходы от слезливости до «веселья». В состоянии приподнятого настроения может раздавать свои вещи, предлагать деньги со своей пенсии сиделке. Такое поведение непонятно родственникам, так как всегда отличалась холодностью, черствостью и скупостью. Какое из перечисленных ниже расстройств НАИБОЛЕЕ вероятно?
слабодушие
амбивалентность
гипертимия
эйфория
дисфория
Жалобы дочери 60 летнего человека на проблемы с его поведением. В течение двух лет отмечается нарастающая конфликтность, обидчивость, снижение деятельности, нарушена ориентировка в месте, времени, не может назвать свое имя и домашний адрес. Пренебрегает гигиеническими навыками, не замечает упускания мочи. Требует постоянного надзора и ухода. Какое расстройство НАИБОЛЕЕ вероятно?
деменция вследствие органического поражения wyc
шизофрения старческого возраста
расстройство личности
болезнь Альцгеймера
умственная отсталость
Мужчина 32 лет, после контакта с кошкой через 2 часа появилось удушье, свистящее дыхание с удлиненным выдохом. Ранее при контакте с кошкой появлялся зуд глаз, носа, чихание, слезотечение, ринорея и кашель. Какой препарат позволит купировать данное состояние?
сальбутамол
дезлоратадин
ксимтазолин
кетотифен
монтелукаст
.Мужчина 65 лет страдает артериальной гипертензией, изредка принимает энам в таблетках. После стресса, час назад появилась резкая головная боль, ноющие боли в области сердца, рвота, головокружение, «туман в глазах», сужение полей зрения, слабость в правых конечностях, сухожильные рефлексы справа не вызываются. АД 220/110 мм.рт.ст. На ЭКГ, синусовый ритм, ЧСС 60 в мин. В RV5 >RV4 косо-нисходящее смещение ST с от изолинии, зубец T отрицательный. Какой диагноз?
Гипертонический криз, церебральный вариант
Гипертонический криз осложненный отеком сетчатки
Гипертонический криз, ОНМК
Гипертонический криз, отек мозга
Гипертонический криз, острая энцефалопатия
На прием к врачу обратился мужчина 48 лет,. После стресса, 4 часа назад резко усилилась головная боль, появился шум в ушах. При осмотре: лицо гиперемировано, очаговой неврологической симптоматики нет. В легких везикулярное дыхание. АД 160/100 мм.рт. ст., тоны сердца ритмичные. ЧСС-70 в мин. Акцент 2 тона на аорте. Назначение какого лекарственного препарата НАИБОЛЕЕ целесообразно для неотложной помощи?
+Каптоприл
Небетолол
Валсартал
Бисопролол
Нитроглицерин
После ДТП спустя несколько месяцев, водитель, совершивший наезд на ребенка, испытывает тревожность, страхи, пониженное настроение, кошмарные сновидения. Стал избегать места, где бывает скопление детей, однажды, «кажется, увидел девочку, которую сбил». В звуке тормозов машин, может слышать крик девочки. Относится к этим переживаниям критично. Какое из ниже перечисленных расстройств НАИБОЛЕЕ вероятно?
посттравматическое стрессовое расстройство
органическое (посттравматическое) заболевание головного мозга
обсессивно-компульсивное расстройство
конверсионное расстройство
депрессивная фаза МДП
Пациент, имеющий хронический эндогенный процесс находится в приподнятом настроении, говорит много, практически без остановок. Структура речи формально не нарушена, грамматических ошибок нет, но предложения не несут никакого смысла, нет связи между словами. Какое это нарушение?
шизофазия
бред
онейроид
резонерство
помрачение сознания
.У пациента с тяжелой пневмонией нарастает одышка и тахикардия, а АД упало до 80/40 мм рт.ст. Диагностирован инфекционно-токсический шок. Начата инфузионная терапия. Какой препарат НАИБОЛЕЕ целесообразно добавить при интенсивной терапии шока?
Допамин
Адреналин
Атропин
Корглюкон
Гепарин
25-летний мужчина обратился по поводу учащенного сердцебиения, потливости в течение 6 месяцев. Симптомы появляются, когда он выступает перед большой аудиторией. Во время выступлений его мысли сбиваются, он боится, что он покраснеет и все будут смеяться над ним. Часто курит марихуану, чтобы успокоить нервы. В анамнезе персистирующая астма, принимает сальбутамол по необходимости. Не пьет алкоголь. Кардиопульмональное обследование не показывает никаких отклонений. Пациент выглядит взволнованным. Что из перечисленного является следующим шагом в управлении заболеванием?
Когнитивно-поведенческая терапия
Лоразепам терапия
Терапия пропранололом
Назначение ГКС
Назначение бензодиазепинов
Больной 22 лет, страдающий бронхиальной астмой, вызвал врача на дом. Объективно: возбужден, температура тела 36,7°С, ЧСС - 120 ударов в мин., число дыханий - 24 в мин. При аускультации: дыхание резко ослаблено, единичные сухие хрипы. Из анамнеза: в течение суток получил более 10 ингаляций беротека без эффекта. С какого препарата следует начать оказание неотложной помощи?
Метилпреднизолон
Тиосульфат натрия
Эуфиллин
Сальбутамол
Будесонид
Девушка 16 лет, студентка колледжа. Потеряла сознание во время собрания в душном помещении на 1-1,5 минуты. Спутанности сознания после обморока не было. Перед обмороком почувствовала тошноту. Отрицает заболевания сердца, прием лекарственных препаратов, беременность. ЧСС 86 уд/мин, АД 110/70 мм рт.ст. Какая дальнейшая тактика наиболее целесообразна?
Снять электрокардиограмму
Оставить под наблюдением на 24 часа
Девушка здорова, отправить домой
Необходима консультация невропатолога
56-летний мужчина обратился к врачу для последующей экспертизы после биопсии височной артерии, которую проводили для оценки головной боли и расплывчатого зрения. В анамнезе – артериальная гипертензия в течение 12 лет, принимает лизиноприл. Результаты тестирования показывают снижение концентрации аргинина в эндотелиальных клетках сосудов. Что из перечисленного наиболее вероятно объясняет полученный результат?
Увеличение производства оксида азота
Увеличение фосфорилирования миозина
Снижение концентрации брадикинина
Увеличение связывания кальция
Активация миозинкиназы
38-летний мужчина, чабан, житель Аральского района, Кызылординской области. Заболел остро, отмечает повышение температуры тела, слабость, головную боль, болезненное образование в правой паховой области. Температура тела 40,00С. В паховой области справа увеличенный лимфоузел, болезненный, неподвижный, с нечеткими границами, кожа над ним гиперемированы. В области передней поверхности правой голени пустула, наполнена темно-кровавым содержимым, болезненная. Вас вызвали к пациенту на дом. Ваша тактика?
в первую очередь сообщить о случае чумы по цепочке экстренного оповещения
в первую очередь надеть противочумный костюм
немедленно начать детоксикационную терапию
вызвать бригаду скорой помощи на себя
начать экстренную химиопрофилактику всех контактных лиц
.Пациент 17 лет, при лечении в дневном стационаре ему сделали цефазолин в/м. Через 5 минут у больного началась выраженная одышка, отек слизистой рта и языка, бледность, потливость. Пульс слабый нитевидный. Какой препарат НАИБОЛЕЕ целесообразно ввести пациенту на первом этапе оказания неотложной помощи?
Адреналин
Димедрол
Триамциналон
Супрастин
Допамин
52-летний мужчина доставлен в приемный покой с внезапным началом генерализованной головной болью в течение 2 часов. Появилась тошнота, была рвота. Симптомы начались внезапно, когда он был дома. 6 дней назад у него была сильная головная боль, которая прошла самостоятельно Курит по 2 пачки сигарет ежедневно в течение 30 лет. Принимает лизиноприл, симвастатин. PS82/мин, АД 160/100 мм рт ст. зрачки ровные, круглые, реагируют на свет. Черепные нервы II-XII интактны. КТ головы не показывает никаких отклонений. Какой шаг в управлении заболеванием является наиболее подходящим?
назначить люмбальную пункцию
повторить КТ через 24 часа
администрирование кислорода
назначить нитропруссид натрия
ограничить потребление соли
Мужчина, 36 лет, обратился в поликлинику с жалобами на сильно распухшую левую кисть, повышение температуры, общую слабость в течение 3-х дней. Неделю назад участвовал в разделке туши коровы, поранил руку. При осмотре: на тыльной поверхности левой кисти безболезненная язва с плотным струпом черного цвета в центре, вокруг слившиеся дочерние пузырьки с серозным содержимым, окруженные венчиком гиперемии в виде валика, выраженный отек левой кисти. Какой диагноз из перечисленных ниже НАИБОЛЕЕ вероятен?

Сибирская язва, кожная форма
Туляремия, кожная форма
Пастереллез, кожная форма
Лейшманиоз, кожная форма
Чума, кожная форма
Девочка 8 месячного возраста осмотрена участковым врачом. Со слов матери жалобы на повышение температуры тела до 39С, вялость, отказ от груди, рвоту. Сегодня, на 4-й день болезни присоединилась желтуха, ребенок стал резко возбужденным, отмечались судороги, рвота типа "кофейной гущи". В анамнезе: 3 месяца назад ребенку проводилось переливание крови. Общий билирубин 165 мкмоль/л, прямая фракция - 144 мкмоль/л, АЛТ - 5,8 мкмоль/л, АСТ - 3,7 мкмоль/л, тимоловая проба -12 Ед. Наиболее первоочередные действия участкового врача в данной ситуации.
передать экстренное извещение, больную срочно госпитализировать
срочно маркерная диагностика, затем госпитализация
срочно ОАК, затем госпитализация
передать экстренное извещение, до получения результатов маркерной диагностики наблюдать на дому
передать экстренное извещение, срочно кровь на маркерную диагностику
59-летняя женщина обратилась к врачу из-за постепенного ухудшения координации и непроизвольных движений в левой руке в течение последних 6 месяцев. Ее муж сообщает, что она стала замкнутой и апатичной за этот период. Она ориентирована на время, место и личность. Исследование показывает бимануальный, ритмичный, низкочастотный тремор, который более выражен в левой руке. В руках и ногах нормальный диапазон движений: активный движения очень медленные. Мышечная сила в норме, повышенное сопротивление пассивному сгибанию и разгибанию конечностей. Ходит маленькими шагами и шатающейся походкой. Какой диагноз наиболее вероятен?
Болезнь Паркинсона
Болезнь Вильсона
Печеночная недостаточность
Гипертиреоз
Ишемия мозжечка
Мужчина 47 лет, обратился с жалобами на боли в шейном отделе позвоночника, слабость в кистях рук больше слева, скованность в ногах, неуверенность при ходьбе, недержание мочи. В анамнезе: неделю назад в результате ДТП получил травму шейного отдела позвоночника, за медицинской помощью не обращался.
Объективно при осмотре: выраженная болезненность и напряжение мышц шеи. Тонус мышц в нижних конечностях повышен по пирамидному типу. Сухожильные рефлексы с рук снижены, с ног высокие, симптом Бабинского с 2х сторон. Проводниковые нарушения чувствительности с уровня С6, С7. Нарушена функция тазовых органов. Какой НАИБОЛЕЕ вероятный диагноз?
цервикальная радикуломиелопатия
сирингомиелия
опухоль спинного мозга
боковой амиотрофический склероз
спинальная амиотрофия
Вы наблюдаете пациента, молодого мужчину с жалобами на лихорадку в течение последнего месяца, похудание на 8 кг за это же время, диарею (стул 2-6 раз в сутки), при осмотре были выявлены увеличенные подключичные, паховые и подмышечные лимфоузлы, то о каком заболевании в первую очередь вы подумаете?
ВИЧ-инфекция
Болезнь Крона
Сепсис
Хроническая дизентерия
Болезнь Уиппла
.Ранее здоровая 18-летняя женщина обратилась по поводу жидкого стула и спазмов в животе с предыдущего вечера. У нее было 3-4 эпизода жидкого водянистого стула. Ее тошнит и дважды была рвота, ела 2 дня назад. Она была на веганской диете в течение 6 месяцев. Лекарств не принимает, никуда не ездила. Температура тела 36,8ᴼС, пульс 73/мин, АД 110/70 мм рт ст. При осмотре – сухость слизистых оболочек. Что вызвало данное состояние?
Норовирус
Иерсинии
Шигеллы
Золотистый стафилококк
Сальмонеллы
У ребенка 3-х лет в течение нескольких недель рвота, периодические судороги, высокая температура, сопорозен. Из анамнеза: контакт с больной туберкулезом тетей, под наблюдением фтизиатра не был. Не привит БЦЖ в связи с мед.отводом. Какое из перечисленных обследований врач обязан провести в первую очередь для уточнения диагноза?
пинномозговую пункцию
Рентгенографию легких
Общий анализ крови и мочи
Копрограмму и бакпосев кала
Внутрикожную пробу Манту и Диаскинтест
У студента 20 лет, при профилактическом флюорографическом обследовании в S1 правого легкого обнаружены очаги с нечеткими контурами малой и средней интенсивности. Беспокоит периодически кашель со скудной мокротой. Из анамнеза: курит в течение 5 лет. Предыдущее флюорографическое обследование в прошлом году без патологических изменений. МБТ в мокроте не обнаружены. Какой наиболее вероятный диагноз?
Очаговый туберкулез легких
Хронический бронхит
Очаговая пневмония
Периферический рак легкого
Бронхолобулярный инфильтрат
72-летняя женщина обратилась по поводу лихорадки, миалгии, кашля в течение 3 дней. Живет во времянке, у соседей были похожие симптомы. В анамнезе – АГ, принимает лизиноприл. Температура тела 38,9ᴼС, пульс 105/мин, ЧДД 22/мин, АД 110/60 мм рт ст., SаO2 89%. В крови – лейкоцитов 10 500/мм3, креатинин – 0,9 мг/дл, прокальцитонин 0,05 мкг/л (норма<0,06). На R легких – двустороннее потемнение в нижних долях. Начато лечение цефтриаксоном и азитромицином. Через два дня - температура тела 37,6ᴼС, прокальцитонин 0,04 мкг/л, SаO2 96%. Что является подходящим шагом в управлении заболеванием?
Прекратить цефтриаксон и азитромицин
Продолжать цефтриаксон и азитромицин до 7 дней
Повторная рентгенография легких
Прекратить цефтриаксон и продолжить азитромицин
Посев мокроты
51-летний мужчина на приеме, 9 месяцев назад перенес острый вирусный гепатит В. При осмотре – патологии не выявлено. Исследование сыворотки крови – повышенная печеночная трансаминазная активность, вирусная нагрузка гепатита В 4286 МЕ/мл. Какие результаты ожидаемы у пациента?
HBeAg +, антигемоглобин -, Anti-HBc IgG -, Anti-HBc IgМ+
HBeAg -, антигемоглобин +, Anti-HBc IgG +, Anti-HBc IgМ-
HBeAg -, антигемоглобин -, Anti-HBc IgG +, Anti-HBc IgМ-
HBeAg -, антигемоглобин +, Anti-HBc IgG -, Anti-HBc IgМ-
HBeAg -, антигемоглобин -, Anti-HBc IgG -, Anti-HBc IgМ+
.Проводится профосмотр в детском саду перед школой. Девочка 6 лет. От 1 доношенной беременности и родов, протекавших без патологии. Растет и развивается соответственно возрасту. Перенесенные заболевания – перинатальная энцефалопатия до 1 года, атопический дерматит-после 3-х лет. На момент осмотра состояние удовлетворительное. Физическое и психомоторное развитие соответствует возрасту. По внутренним органам без особенностей. Какую медицинскую документацию необходимо заполнить ребенку?
форма 026/у, 063/у, паспорт здоровья
форма 063/у, паспорт здоровья
форма 026/у, 112/у, паспорт здоровья
форма 112/у, 063/у, паспорт здоровья
форма 026/у, паспорт здоровья
Пациент 22 лет, обратился к терапевту с жалобами на повышение температуры до 37,7о по вечерам, потливость, слабость, недомогание, кашель с мокротой около 2-х недель, боль в грудной клетке справа, потерю веса. Какое мероприятие из перечисленных является наиболее целесообразным?
Исследование мокроты на МБТ, обзорная рентгенограмма
Бронхоскопия, обзорная рентгенограмма
Спирография, обзорная рентгенограмма
Спирография. Бронхоскопия
УЗИ плевральной полости, рентгенография
Ребенку в возрасте 1 года 4 месяцев поставлена проба Манту – папула 8 мм. Контакт с больным туберкулезом не установлен. Жалоб нет. Симптомы интоксикации отсутствуют. Диаскинтест – отрицательный. Какой наиболее вероятный предварительный диагноз в данном случае?
Поствакцинальная аллергия
Парааллергическая реакция
Инфекционная аллергия
Параспецифическая реакция
Токсико-аллергическая реакция
.Мама ребенка 4-х лет, обратилась с жалобами на общее недомогание, субфебрильную температуру, частые ОРВИ. Из анамнеза: установлен однократный контакт с больным активным туберкулезом дядей. Объективно: состояние средней тяжести, умеренные симптомы интоксикации, температура - 37,5 0. Бледные кожные покровы, пониженного питания. Физикальные данные – без патологии. На рентгенограмме: негомогенное затемнение в верхней доле правого легкого, связанное дорожкой с увеличенным трахеобронхиальным лимфоузлом. Какое из перечисленных исследований необходимо провести в первую очередь для уточнения диагноза?
Проба Манту и Диаскинтест
Микроскопия и бакпосев мокроты на МБТ
Компьютерная томография легких
Общий и биохимический анализ крови и мочи
ИФА и ПЦР диагностика
Ребенок 1 год, привит в родильном доме БЦЖ, рубчика на левом плече нет. Поставлена проба Манту – папула 13 мм. Определите характер туберкулиновой чувствительности?
Первичное инфицирование МБТ
Поствакцинальная аллергия
Аллергическая реакция на туберкулин
Заболевание туберкулезом
Латентный микробизм
.Ранее здоровый 3-летний мальчик был доставлен родителями к врачу из-за лихорадки и сыпи в течение 6 дней. Температура тела 38,9ᴼС. При осмотре – правосторонняя передняя шейная лимфаденопатия, двухсторонняя инъекция конъюнктивы, эритема языка и губ, на коже рук, ног, промежности, туловища – макулопапулезная сыпь. Какое наиболее распространенное осложнение может развиться у пациента?
Аневризма коронарной артерии
Быстропрогрессирующий гломерулонефрит
Потеря слуха
Ретинопатия
Невропатия
К врачу общей практики на скрининговый осмотр пришел молодой мужчина М., 37 лет. Жалоб не предъявляет. Курит. В анамнезе: отец болен ишемической болезнью сердца и АГ. По результатам физикального осмотра ИМТ - 34, периодическое повышение артериального давления до 135-140/85 мм РТ ст. К какой группе динамического наблюдения относится этот пациент?
Д-IБ
Д-IА
Д-II
Д-III
Д-IV
Подросток 15 лет, обратился по поводу фурункулеза. Из анамнеза: предрасположен к простудным инфекциям, старший брат болеет сахарным диабетом. Объективно: физическое развитие на 7 лет. Вес 40 кг при росте 150 см. Кожные покровы бледные, сухие.
Множественные фурункулы на коже туловища и шее. АД 105/65 мм.рт.ст. Пульс 88 в мин. Какое обследование в первую очередь является НАИБОЛЕЕ информативным и достоверно позволит подтвердить основной диагноз?
определение гликемического профиля
определение глюкозы в моче
определение лейкоформулы в крови
определение С пептида натощак
бактериологическое исследование
При скриннинге у мужчины 30 лет были выявлены жалобы на жажду, сухость во рту. При осмотре:
Рост 176 см. Вес 84 кг. ИМТ 27,1. В крови: глюкоза натощак 6,7 ммоль/л. Какое обследование должно быть проведено на следующем этапе?
определение гликемического профиля
определение глюкозы в моче
определение лейкоформулы в крови
определение С пептида натощак
липидный профиль
.Мужчина 45 лет, обратился к участковому врачу с жалобами на повышение АД до 145/95 мм рт.ст. Повышенное АД было выявлено при профосмотре однократно. Не курит, вредных привычек нет, наследственность не отягощена. Объективно: ИМТ 32, АД 135/65. Глюкоза - 4.8 ммоль/л, холестерин - 4,8 ммоль/л. Какая рекомендация должна быть дана пациенту?
Пациент должен быть направлен в школу здоровья по АГ
дать рекомендации по ЗОЖ и рекомендовать осмотр через 2 года
дать рекомендации соблюдать ЗОЖ и повторно обследоваться через 6 месяцев
необходимо сделать ЭКГ
направить на консультацию к кардиологу
.Врач общей практики обслуживает вызов на дому. Будучи инвалидом II группы по основному заболеванию суставов, пациент не может быть госпитализирован в стационар и в то же время нуждается в уходе. На какой срок выдается лист временной нетрудоспособности его родственнику, приехавшему из другой местности, для ухода за заболевшим?
на весь период заболевания..
До 7 дней.
до 14 дней через ВКК.
До 3-х дней
выдается справка о временной нетрудоспособности
В поликлинике в течение дня был проведен скрининг 28 взрослых от 40 до 55 лет. В какую форму медицинской документации должны быть внесены результаты скриннинга?
ф. 025-08/у
ф.030/у
ф. 076/у
ф. 080/у
ф. 112/у
.51-летняя женщина обратилась по поводу боли в правой поясничной области и кровавой мочи. Отмечает, что последние 2 недели развился прогрессирующий отек нижних конечностей и увеличился вес на 3 кг. В анамнезе хронический гепатит В, диагностированный 10 лет назад. PS 98/мин, ЧДД 18/мин, АД 135/75 мм рт ст. При осмотре – периорбитальный отек, вздутие живота, отек нижних конечностей. На КТ – узловая печень с асцитом, большая правая почки с коллатеральными сосудами и заполненными дефектами в правой почечной вене. В ОАМ – белок ++++, глюкоза, жир. Что из нижеследующего является вероятной причиной патологии почек?
потеря антитромбина III
приобретенный дефицит фактора VIII
увеличение венозного застоя
нарушение выработки эстрогена
антифосфолипидные антитела
Мужчина 42 лет, при прохождении скриннинга на выявление глаукомы были выявлены следующие показатели внутриглазного давления бесконтактным методом: правый глаз - 25 мм рт.ст, левый глаз - 18 мм рт.ст. Какая дальнейшая тактика?
направить на дообследование офтальмологу
при наличии наследственной предрасположенности направить для дообследования к офтальмологу
рекомендовать повторное обследование через 3 месяца
направить в глаукомный кабинет
направить на дообследование в условиях стационара
Мужчина 35 лет, гражданин Российской федерации. Обратился по поводу ОРВИ. При осмотре состояние средней тяжести, лихорадка 38.0°С, караральные явления. Работает в одной из компаний в Астане. Какой документ о временной нетрудоспособности ему следует выдать?
Лист о временной нетрудоспособности
Невостребованный бланк листа о временной нетрудоспособности
Справка о временной инвалидности
Заключение врачебно-консультативной комиссии
Справка о временной нетрудоспособности
Супружеская пара усыновила ребенка из дома малютки. Ребенок новорожденный, родился недоношенным. Супруга работает на отвественной должности, супруг - работник творческой професии. Они хотят возложить основную заботу о ребенке на приемного отца. Необходимо выдать документ о временной нетрудоспособности. Кому и и какой документ может быть выдан?
любому из приемных родителях, лист о временной нетрудоспособности
только приемной матери, лист о временной нетрудоспособности
только приемному отцу, лист о временной нетрудоспособности
любому из приемных родителях, справку о временной нетрудоспособности
можно приемному отцу, справку о временной нетрудоспособности
Мужчина 45 лет, обратился к участковому врачу с жалобами на преходящие боли за грудиной, которые появились 2 недели назад, длительностью 1-2 мин, проходят спонтанно, возникают при физической нагрузке. Курит 1 пачку в день. Объективно: ИМТ - 32, АД 135/65 мм рт.ст. ЧСС 75 в мин, тоны сердца ясные ритмичные. Какое обследование показано на первом этапе диагностического поиска?
Электрокардиография
Р-графия органов грудной клетки
Эхокардиография
Допплер-сонография сонной артерии
Проба с эргометрином
Женщина 55 лет обратилась с жалобами на головные боли, которые появились несколько недель назад, возникают к концу рабочего дня. При измерении АД в кабинете доврачебного осмотра медсестрой - 145/95 мм рт.ст. Женщина с ИМТ 30, вредных привычек нет, наследственность не отягощена. Какое обследование показано на следующем этапе?
Повторное измерение АД не менее чем через 6 часов
Р-графия органов грудной клетки
Эхокардиография
Электрокардиография
Допплер-сонография сонной артерии
В поликлинику обратился ребенок 6 лет из социально обеспеченной семьи для оформления в школу. Ранее ребенок дошкольные учреждения не посещал. В 3 летнем возрасте перенес острую пневмонию без осложнений, в 5 лет – острую кишечную инфекцию. При осмотре нервно-психическое и физическое развитие соответствует возрасту. Привит согласно календарному плану. Какая первичная медицинская документация сдается в школу?
ф. 063/у
ф. 026/у
ф.030/у
ф. 076/у
ф. 080/у
57-летний мужчина с ВИЧ и ГЭРБ обратился в связи с кашлем с продуктивной мокротой и ночные поты, появившиеся на прошлой неделе. Отмечает, что мокрота с неприятным запахом. Температура 38,9°С. При осмотре – грубый треск и тупость при перкуссии у правого основания легкого. Рассеянные хрипы на выдохе по обоим легочным полям при аускультации. На рентгене –(смотрите снимок) Что является причиной симптомов у данного пациента?

аспирационная пневмония
бронхоэктазия
пневмоцистная пневмония
туберкулез
аденокарцинома
Женщина 27 лет на сроке беременности 6-7 недель обратилась с целью искусственного прерывания беременности. Две недели назад пересла ОРВИ в тяжелой форме, по поводу которого лечилась амбулаторно. Женщина является гражданкой РК и имеет постоянную работу. Какой документ следует ей выдать?
Лист о временной нетрудоспособности
Невостребованный бланк листа о временной нетрудоспособности
Справка о временной инвалидности
Заключение врачебно-консультативной комиссии
Справка о временной нетрудоспособности
9.Студент одного из вузов, находясь на производственной практике обратился с жалобами на тошноту, рвоту, недомогание. Был выставлен диагноз "Острая пищевая токсикоинфекция". Какой документ о временной нетрудоспособности ему следует выдать?
Лист о временной нетрудоспособности
Невостребованный бланк листа о временной нетрудоспособности
Справка о временной инвалидности
Заключение врачебно-консультативной комиссии
Справка о временной нетрудоспособности
10.Мужчина, будучи в состоянии алкогольного опьянения получил перелом лучевой кости. Первую помощь была ему оказана в травмпункте. Какой документ о временной нетрудоспособности ему следует выдать?
Справка о временной нетрудоспособности
Лист о временной нетрудоспособности
Невостребованный бланк листа о временной нетрудоспособности
Справка о временной инвалидности
Заключение врачебно-консультативной комиссии
11.Женщина 39 лет, мать-одиночка, воспитывает 3 детей дошкольного возраста, болеет туберкулезом. Не имеет возможности получать лечение в дневном стационаре городской поликлиники. Лечение оказывается в условиях стационара на дому мобильной бригадой. Какова частота визитов фтизиатра для контроля лечения пациентки?
выезжает с мобильной бригадой 1 раз в 10 дней
выезжает с мобильной бригадой ежедневно
выезжает с мобильной бригадой 1 раз в 7 дней
выезжает с мобильной бригадой 1 раз в 14 дней
выезжает с мобильной бригадой 1 раз в месяц
12.Мужчина 23 лет, с исходным бактериовыделением, начал лечение против туберкулеза в круглосуточном стационаре. Родители просят перевести его на амбулаторное лечение. При каком условии возможен перевод на амбулаторное лечение?
при двукратном отрицательном результате микроскопии
письменный запрос родителей
согласие пациента
разрешение поликлиники
при отрицательном результате микроскопии
13.У беременной 27 лет на сроке 36 недель отмечается повышение АД до 170/100 мм рт.ст., увеличение массы тела на 4 кг за неделю, отеки на нижних конечностях, снижение суточного диуреза до 400 мл. В крови: тромбоциты – 90 тыс, альбумины – 16 г/л. В моче: белок – 2,9 г/л. На УЗИ: задержка развития плода на 4 недели. Какая тактика ведения беременной НАИБОЛЕЕ целесообразна?
Досрочное родоразрешение
Пролонгирование беременности до самопроизвольного родоразрешения
Пролонгирование беременности с антигипертензивной терапией
Прерывание беременности с антигипертензивной терапией
Антигипертензивная терапия
15.Женщина 32 лет. Беременность 26-27 недель. Беременность первая. При очередном обследовании выявлена гликемия 6.2 ммоль/л. Обследование 2 года назад гликемия - 4,8 ммоль/л. Какой прогноз для ребенка наиболее вероятен в данном случае?
высокий риск крупного плода
вероятность задержки внутриутробного развития
высокий риск развития ВПС
вероятность веса меньше 2500 г при рождении
высокий риск преждевременных родов
16. Пациентка 26 лет, беременная в сроке 22-23 недели обратилась к врачу общей практики (ВОП) с жалобами на головную боль, тошноту, рвоту, боли в правом подреберье и снижение мочеотделения. При объективном осмотре пастозность нижних конечностей, АД 150/90 мм рт ст на обеих руках, пульс 96 ударов в одну минуту. Выберите гипотензивный препарат для данной пациентки?
Метилдопа.
Каптоприл.
Гипохлортиазид
Платифиллин
Бисопролол
17.Женщина 27 лет, во время беременности при возникновении угрозы прерывания беременности был назначен дюфастон (дидрогестерон). Через 2 недели после приёма дюфастона отмечались кожный зуд, повышение показателей трансаминаз (АЛТ – 885 ед/л, АСТ – 447 ед/л), которые снизились после отмены препарата. Родила здорового ребенка. Какие рекомендации по контрацепции целесообразны в послеродовом периоде?
контрацепция барьерными средствами
контрацепция мини-пили
контрацепция дезагистрел
контрацепция левоноргистрел
контрацепция депо-проверой
18.Женщина 32 лет, родила здоровую девочку, вес 3200г, рост 50 см. Девочку вакцинировали. Муж женщины болен туберкулезом, бактеривыделение . Живут в однокомнатной квартире. На какой срок вакцинированные новорожденные дети, попадающие из перинатального (родильного отделения) центра в условия контакта с больным бактериовыделителем, в случае невозможности изоляции больного туберкулезом, изолируются в отделения выхаживания новорожденных или дома ребенка?
на 10 дней
не менее чем 2 месяца
на 2 недели
на 1 месяц
на 21 день
19.У 34-летней женщины жалуется на зуд, выделения из влагалища с неприятным «рыбным запахом». На зеркалах шейка матки - внешний вид не изменен. Какое из следующего, скорее всего, соответствует этиологии данного состояния?
Бактериальный вагиноз
Neisseria gonorrhea
Chlamydia trachomatis
Кандидозный вагинит
Staphylococcus aureus
21.39-летняя женщина обратилась к врачу из болезненных тяжелых менструаций, которые беспокоят ее в течение 5 месяцев. Ранее менструации были нормальными и проходили с 28-дневным интервалом и длились в течение 3-х дней. Теперь они длятся 7-8 дней, обильные, со сгустками. При обследовании органов малого таза – матка вялая, равномерно увеличена, соответствует 8-недельной беременности. Тест мочи на беременность отрицательный. Что является вероятной причиной данного состояния у пациентки?
эндометриальная ткань в стенке матки
полип матки
карцинома эндометрия
аномальное утолщение ткани эндометрия
доброкачественная опухоль миометрия
22. Женщина 28 лет. Месяц назад родила в срок здорового ребенка весом 3300 г, рост 51 см. Особых жалоб не предьявляет, лактация хорошая, ребенок полностью на грудном вскармливании. При осмотре обращает внимание бледность слизистых, при расспросе - появилась слоистость ногтей, волосы потускнели и стали больше выпадать. Питается удовлетворительно, никаких дополнительных препаратов (поливитаминов и прочее) не принимает. Принимала "Прегнавит" во время беременности, но посчитала что после родов необходимость в витаминах и микроэлементах отпала. В крови - гемоглобин 106 г/л, ЦП - 0,7. В течение какого периода рекомендуется назначение железа в дозе не менее 60 мг в сутки родильницам?
в течение шести месяцев после родов
в течение первого месяца после родов
в течение трех месяцев после родов
до введения первого прикорма
не менее 12 месяцев после родов
23.Женщина 22 лет, недавно начала регулярную половую жизнь. Обратилась с жалобами на светлые скудные выделения и неприятные ощущения во время полового контакта. Обследование на ЗППП выявило антитела к Chlamydia trachomatis IgM. ПЦР материала из цервикального канала подтвердил наличие Chlamydia trachomatis. У ее полового партнера - ИФА на антитела к Chlamydia trachomatis дали отрицательный результат. Какая тактика наиболее целесообразна?
курс азитромицина, лечить обоих половых партнеров
курс азитромицина, лечить только женщину
левофлоксацин, лечить обоих половых партнеров
курс ровамицина, пролечить женщину, ее партнеру повторить ИФА через 3 месяца
курс клиндамицина, ее партнера проверить методом ПЦР
25.Женщина 56 лет.Обратилась с жалобами на одышку при небольшой физической нагрузке, сухой кашель. Одышка носит смешанный характер. Состоит на Д учете с диагнозом ХОБЛ в течение 5 лет. Курит более 30 лет по 1-1,5 пачки в день. Больная отмечала появление прожилок крови при приступах надсадного малопродуктивного кашля. Отмечает что и раньше была одышка, но теперь она сильнее и стало более трудно вдохнуть, нежели выдохнуть. Р-грамма ОГК не выявила существеннных различий с ренген-снимками прошлого года. В анализе крови - СОЭ 54 мм/ч. Какой обследование необходимо провести в первую очередь?
Бронхоскопию с биопсией
КТ органов грудной клетки
Спирографию
определение онкомаркера SYFRA
GenExpert
26.Женщина 34 лет. Стоит на учете с диагнозом "Системная красная волчанка". Ей планируется поведение биологической терапии. Ей необходим надежный метод контрацепции. Какой оптимальный метод контрацепции в данном случае?
внутриматочная спираль с медью
внутриматочная спираль с гормонами
чистые прогестиновые контрацептивы
комбинированные оральные контрацептивы
спермициды
27.Женщина 29 лет, через 6 месяцев после рождения второго ребенка, кормит грудью, хочет надежную долговременную контрацепцию. Из анамнеза: хронический гастрит. Каков наиболее оптимальный метод контрацепции в данном случае?
внутриматочная контрацепция
лактационная аменорея
чистые прогестиновые контрацептивы
комбинированные оральные контрацептивы
барьерный метод
28.Женщина 25 лет, через 6 месяцев после рождения первого ребенка, ребенок на смешанном грудном вскармливании, хочет надежной контрацепции на следующие 2 года. Из анамнеза: хронический сальпингоофорит. Каков наиболее оптимальный метод контрацепции в данном случае?
чистые прогестиновые контрацептивы
лактационная аменорея
внутриматочная контрацепция
добровольная хирургическая стерилизация
комбинированные оральные контрацептивы
29.Девушка-подросток, 16 лет, страдает себореей (увеличение сальности кожи), акне, повышенное оволосение на руках, ногах, передней брюшной стенке. АД -111/74 мм рт ст., вес 45 кг, рост 152 см, ИМТ-19,5. Какой гестагенный компонент в составе гормональных контрацептивов наиболее приемлем в данном случае?
дроспиренон
левоноргестрел
дезогестрел
гестоден
норгестимат
30.Девушка-подросток, 18 лет имеет сексуально активную жизнь и часто меняет сексуальных партнеров. Какой метод контрацепции наиболее приемлем ей?
Презерватив
Биологические методы
Комбинированные оральные контрацептивы
Внутриматочная спираль
Диафрагма
31.Ранее здоровая 36-летняя женщина обратилась из-за прогрессирующей головной боли в течение 5 дней. Не курит, алкоголь не употребляет. Сексуально активна с одним партнером, принимает оральные контрацептивы. Т тела 37,5ᴼС, пульс 105/мин, АД 125/80 мм рт ст. При обследовании – разрыв правого глаза. Зрачки равны, реагируют на свет. При фундоскопии – двусторонний отек диска зрительного нерва. Что из нижеследующего является наиболее подходящим в управлении заболеванием?
введение низкомолекулярного гепарина
анализ спиномозговой жидкости
введение антибиотиков
определение уровня D-димера
эндоваскулярный тромболизис
33.43-летний мужчина ВИЧ-инфицированный обратился в связи с лихорадкой и ночными потами в течение 15 дней, головными болями, слабостью. Получает противовирусную терапию. Состоит на учете в наркодиспансере, курит, пьет алкоголь. Узелки на подушечках пальцах, кровизлияния под ногтями, кровоизлияния в конъюктиву нижнего века. Пансистолический шум вдоль левого края грудины, усиливается на вдохе. АД 130/80 мм рт ст. ОАК: лейкоциты 12.8х109/л, СОЭ 52 мм/ч. Что является наиболее вероятным осложнением состояния?
Легочная эмболия
Кровоизлияния под ногтями
Узелки на пальцах
Ретинальные кровоизлияния
Гематурия
34.Женщина 80 лет. Отмечает снижение массы теа за последние шесть месяцев на 7 кг, появления отвращения к мясу. В анамнезе - хронический гастрит. Объективно: бледность кожных покровов, пальпация живота безболезненная. В крови выявлена анемия 2 степени, СОЭ 48 мм рт.ст. Какое обследование необходимо провести для уточнения диагноза?
ФЭГДС
Р-графию органов грудной клетки
Уреазный дыхательный тест
Трепанобиопсию костного мозга
СЕА и СА19-9
35.Мужчина 45 лет. Жалобы на изжогу, боли и дискомфорт за грудиной в течение ряда лет. При ФГДС были выявены участки метаплазии эпителия (фото). С какой периодичнотью больному необходимо проводить контрольное обледование для раннего выявления возможного осложнения?

1 раз в три месяца
1 раз в месяц
1 раз в год
1 раз в два года
1 раз в три года
36.Больной 74 лет получал антибактериальную терапию по поводу пневмонии. В динамике состояние без улучшения, нарастает слабость, похудание, боли в грудной клетке. При контрольной компьютерной томографии - очаг в размерах несколько увеличился, высокой плотности с неровными контурами. Какой результат исследования мокроты НАИБОЛЕЕ вероятен?

Атипичные клетки
Грам-отрицательные кислотоустойчивые бациллы
Нейтрофильный цитоз
Грибковый мицеллий
Гантелевидные тельца
37.Через 2 дня после родоразрешения у 23-летней женщины появилась прогрессирующая пульсирующая боль в затылке. Боль распространяется в область шеи и плеч. Была тошнота и один эпизод рвоты. Женщина хочет находиться в темной и тихой комнате. симптомы усугубляются, когда она встает, чтобы пойти в ванную и слегка улучшаются, когда она лежит. перенесла эпидуральную анальгезию с адекватным обезболиванием. Неврологическое обследование - патологии не выявлено. Что является наиболее подходящим следующим шагом в управлении заболеванием?
«Заплата» (эпидуральная инъекция крови) в месте эпидуральной инъекции
Назначение аспирина
КТ-ангиография
Строгий постельный режим
Анализ спинномозговой жидкости
41.48-летняя женщина обратилась по поводу боли в грудной клетке. Она описывает боль как ощущение сдавливания с отдачей в левую лопатку. Эпизод начался 15 минут назад, когда она сидела в очереди на прием. Пациентка испытывала такую боль раньше, обычно по вечерам, проходили в покое. PS 112 ударов/мин, АД 121/81 мм рт ст. Тропонины сыворотки отрицательны на двух последовательно взятых анализах крови. ЭКГ без патологии.
Что из нижеперечисленного является лучшим для долгосрочного лечения симптомов данного пациента?
дилтиазем
клопидогрель
аспирин
эналаприл
метопролол
42.16-летний мальчик профессионально занимается баскетболом, родители здоровы. Внезапно упал без сознания во время игры в баскетбол. Через 20 минут был доставлен в отделение скорой помощи, пульса нет, дыхания нет. Врач констатировал внезапную смерть. Родители согласились на вскрытие.
Что вероятнее всего обнаружится на вскрытии?
Гипертрофия межжелудочковой перегородки
Миксоидная опухоль сердца
Дефект межпредсердной перегородки
Эмбол в легочной артерии
Увеличенная нисходящая аорта
48.Мужчина 48 лет, с жалобами на загрудинный дискомфорт, возникающий после эмоциональных перенапряжений. По результатам ЭХОКГ ФВЛЖ (фракция выброса левого желудочка) составила 47%.
Какое исследование (класс I, уровень В) необходимо в обязательном порядке провести пациенту для подтверждения диагноза?
Стресс-эхокардиографию
Позитронно-эмиссионная томография
Коронарную ангиографию
МРТ сердца
Однофотонную эмиссионную компьютерную томографию
50.Какая группа препаратов (класс I, уровень А) из нижеперечисленных рекомендована для медикаментозной терапии всем пациентам со стабильной стенокардией для профилактики «событий»?
Статины
β-блокаторы
Блокаторы кальциевых каналов
Прологированные нитраты
Короткодействующие нитраты
51.Какая группа препаратов (класс IIb, уровень В) из нижеперечисленных рекомендована в качестве препаратов второй линии для медикаментозной терапии всем пациентам со стабильной стенокардией для облегчения симптомов стенокардии/ишемии?
Триметазидин
β-блокаторы
Блокаторы кальциевых каналов
Пролонгированные нитраты
Короткодействующие нитраты
54.42-летняя нерожавшая женщина приходит к врачу для ежегодного гинекологического осмотра и цитологического мазка. Последний год она чувствовала себя здоровой. Использует внутриматочную спираль с медью для контрацепции. Курит 1 пачку сигарет ежедневно в течение 20 лет. Рост 160 см, вес 88 кг, ИМТ 34.4 кг/м2. При бимануальном осмотре – неправильно увеличенная матка. Трансвагинальное УЗИ – гипоэхогенная масса 4,0 см внутри миометрия матки. Что является вероятной причиной этого состояния?
Лейомиома
Рак эндометрия
Аденомиоз
Гиперплазия эндометрия
Полип эндометрия
55.24-летний мужчина пришел на прием по поддержанию здоровья. В анамнезе СД 1 типа, принимает инсулин. Последние 5 лет курил 1 пачку сигарет в день. Не получал никаких прививок. Нормостенического телосложения. ИМТ 23 кг/м2, Т тела 36,2ᴼС, PS 72/мин, ЧДД 35/мин, АД 123/82 мм рт ст.
Какое диагностическое исследование необходимо провести?
Рентгенография грудной клетки
Микроскопическое исследование мокроты
Выполнить ПЦР мокроты
Бакпосев мокроты
Пробное лечение рифампицином
56.65-летняя женщина обратилась по поводу увеличения пальцев в течение 18 месяцев, охриплость голоса в течение 2 месяцев. Снизился аппетит после инфекции дыхательных путей 3 месяца назад и потеря веса на 8 кг в этот период. Никогда не курил. Был выставлен диагноз обструктивного апноэ во сне 10 лет назад. Осмотр – увеличение выпуклости ногтей, болезненный отек мягких тканей пальцев рук и голеностопных суставов.
С чем связано состояние пациента?
Миоз
Увеличение антидиуретического гормона
Периферический цианоз
Увеличение сывороточного кальция
Уменьшение антидиуретического гормона
61.43-летняя женщина обратилась по поводу лихорадки, тошноты и непродуктивного кашля в течение 7 дней. В течение этого периода у нее были головные боли, утомляемость, боли в мышцах и суставах, усиление одышки в течение 2 дней. В анамнезе СД 2 типа и остеортрит левого коленного сустава. Принимает инсулин и ибупрофен. Курила 2 пачки сигарет в день в течение 20 лет, прекратила 10 лет назад. Т тела 38,1ᴼС, PS 94/мин, ЧДД 18/мин, АД 132/86 мм рт ст. Поражение кожи с синим центром, бледной промежуточной зоной и темно-красным периферическим ободком на верхнх и нижних конечностях. Что является вероятным возбудителем заболевания?
Микоплазма
Клебсиелла
Пневмококк
Гемофильная палочка
Золотистый стафилококк
62.50-летний мужчина обратился после падения на улице. Он поскользнулся, ударился о поручень боковой частью груди. Курил в течение 10 лет по 1 пачке сигарет в день, бросил 18 лет назад. Пьет 1-2 банки пива в день. Т тела 37,1ᴼС, PS 78/мин, АД 126/72 мм рт ст. Левая половина грудной клетки болезненна при пальпации. Рентген – 5 мм очаг в правом верхнем легочном поле. Какая тактика наиболее целесообразна?
КТ грудной клетки
Повторить рентген грудной клетки через 12 месяцев
Позитронно-эмиссионная томография
Повторить рентген грудной клетки через 6 месяцев
Анализ высвобождения гамма-интерферона
63.60-летний мужчина проходит осмотр перед холецистэктомией. У матери – хроническое гранулематозное заболевание легких, работает на стекольном заводе. Курит в течение 38 лет по 2 пачки сигарет в день.
На рентгене – впервые выявлен периферически локализованный одиночный легочный узелок.
Какая тактика наиболее целесообразна?
Запросить предыдущую рентгенографию грудной клетки
Провести анализ газов артериальной крови
Выполнить КТ-биопсию
Измерение ангиотензин-превращающего фермента
Выполнить КТ с высоким разрешением
64.43-летняя женщина на обследовании, перед началом работы медсестрой. В течение последнего года у пациентки была легкая одышка и кашель с белой мокротой, особенно по утрам. Вакцинирована по возрасту. Курит по 2 пачки сигарет в день в течение 20 лет и выпивает 1 бокал вина в день. Т тела 36,2ᴼС, PS 74/мин, АД 124/60 мм рт ст. Хрипы по обоим легочным полям. Какое исследование необходимо провести для оценки туберкулеза?
Анализ высвобождения гамма-интерферона
Микроскопию мокроты
Бакпосев мокроты
Рентген грудной клетки
ПЦР мокроты
65.9-летнюю девочку в течение 4-х месяцев беспокоят ранние утренние головные боли и усталость. Головные боли часто сопровождаются тошнотой, иногда бывает рвота, которая приносит облегчение от головной боли. Перестала ходить на занятия балетом 2 недели назад, в связи с тем, что она часто падала. Температура тела 36,8ᴼС, пульс 98/мин, АД 105/65 мм рт ст. У нее широкая походка. Стоя обеими ногами вместе и поднимая руки, закрытие глаз не влияет на равновесие. Не может выполнять быстрые, чередующиеся движения руками. На МРТ - большие (3,5 см), хорошо определены, контрастные кистозные массы в боковой задней ямке без участия желудочков. Какой диагноз наиболее вероятен?
Пилоцетарная астроцитома
Медуллобластома
Диффузная глиома ствола мозга
Эпендиома
Олигодендроглиома
67.Мужчина 55 лет, жалобы на кашель в течение дня с мокротой, одышку, возникающую при быстрой ходьбе, небольшом подъеме. В анамнезе - курит в течение 25 лет по 1 пачке (20 сигарет) в день. Отмечает усиление одышки при простудных заболеваниях в течение последних 5 лет.
Какова степень выраженности одышки по шкале Medical Research Council Dyspnea Scale (MRS)?
степень 1
степень 0
степень 2
степень 3
степень 4
68.Мужчина 45 лет, жалобы на кашель в течение дня, по утрам - с мокротой, одышку, возникающую при ходьбе на 100 м, приходится останавливаться. В анамнезе - курит в течение 25 лет по 1 пачке (20 сигарет) в день.
Какова степень выраженности одышки по шкале Medical Research Council Dyspnea Scale (MRS)?
степень 3
степень 0
степень 1
степень 2
степень 4
70.57-летний мужчина обратился к врачу из-за усиления в течение 2 месяцев одышки при ходьбе. В анамнезе гиперхолестеринемия, принимает симвастатин. В течение 35 лет работал в компании по сносу домов. Курит 33 года по 1 пачке сигарет в день. Выслушиваются мелкие хрипы в обоих легких. На рентгене – диффузные инфильтраты в нижних долях и кальцинированные плевральные бляшки. Какое из состояний разовьется у пациента?
Бронхогенная карцинома
Карцинома щитовидной железы
Туберкулез
Саркоидоз
Мезотелиома
71.59-летняя женщина обратилась по поводу эпизодического кашля и одышки в течение 1 мес. Кашель непродуктивен, усиливается при подъеме по лестнице и ночью. 8 недель назад у нее была температура, боль в горле и заложенность носа. В течение 10 лет АГ. Выкуривает по полпачке сигарет в день ежедневно в течение 16 лет. PS 78/мин, ЧДД 18/мин, АД 145/95 мм рт ст. Пульсоксиметрия 96%. Аускультативно – диффузные хрипы в конце выдоха. Спирометрия: FVC 65% и FEV 60%. Какой диагноз наиболее вероятен?
Бронхиальная астма
Внеболничная пневмония
Хроническая сердечная недостаточность
Дефицит α1-антитрипсина
ГЭРБ
73.62-летний мужчина обратился к врачу в связи с потемнением кожи на носу, в течение 11 месяцев. За последние месяцы пятно увеличилось в размерах. Он фермер. Его мать умерла в возрасте 67 лет от меланомы. При осмотре (см фото). Какой диагноз наиболее вероятен?

Меланома
Кератоакантома
Контагиозный моллюск
Плоскоклеточный рак
Старческий кератоз
74.Сколько времени составляет обработка вызова с момента его получения диспетчером ССМП до передачи для обслуживания бригаде СМП составляет, в течение которого проводится сортировка по категории срочности вызова (Согласно Приказу №450 МЗ РК от 03.07.2017г. «Об утверждении Правил оказания скорой медицинской помощи в Республике Казахстан»)
5 мин
1 мин
2 мин
3 мин
4 мин
до 60 минут
79.Женщина 75 лет, с жалобами на повышение АД до 160/100 мм рт ст. вызвала бригаду скорой медицинской помощи (СМП). По результатам данных осмотра, динамики состояния пациента на фоне проведенных лечебных мероприятий, врачом бригады СМП было принято решение «оставить на дому (по месту проживания)».
По какой форме заполняется врачом бригады ССМП сигнальный лист в случае заболевания пациента и необходимости его посещения на дому участковым врачом?
№ 110-1/у
№ 025/у
№ 114/у
№ 001-3у
№ 003-2/у
80.Сколько времени занимает весь процесс оценки состояния одного пациента согласно медицинской сортировке по Триаж системе?
не более 60 секунд
не более 10 секунд
не более 15 секунд
не более 30 секунд
не более 45 секунд
81.Женщина 65 лет, страдает артериальной гипертензией несколько лет, вызвала бригаду скорой медицинской помощи (СМП) в связи с повышением АД до 160/100 мм рт ст..
Какое время прибытия бригады СМП оптимальное при данной категории срочности?
до 60 минут
до 10 минут
до 15 минут
до 30 минут
до 45 минут
82.69-летняя женщина с острой миелоидной лейкомой обратилась к врачу, чтобы обсудить планы на будущее лечение. Она хочет получить дополнительную информацию по экспериментальной терапии, которую ей предлагают. Врач объяснил механизм действия препарата и предупредил о рисках и преимущества экспериментального лечения. Пациентка заявляет, что она не готова умереть. Врач спрашивает ее, каково ее понимание терапии, она отвечает: «Я не помню деталей, но я хочу попробовать терапию, потому что я не хочу умирать». Какой из этических принципов нарушен во взаимодействии врача с пациентом?
Способность принимать решения
Терапевтическая привилегия
Автономия пациента
Раскрытие информации
Компетенция пациента
83.53-летняя женщина обратилась по поводу нечеткости зрения, головной боли, нескольких эпизодов носовых кровотечений в течение последних нескольких недель. При этом постоянный зуд, особенно по утрам. За последние 6 месяцев – потеря веса на 8 кг, повышенная утомляемость. Температура 37,8°С, АД 160/90 мм рт ст, PS 80/мин, ЧДД 15/мин Травм носа не было. Лицо, ладони, ногтевые ложа, слизистая оболочка полости рта и конъюнктивы выглядят красными. Спленомегалия. В анализах – Hb 19 г/дл, гематокрит – 58%, тромбоциты 450 000/мкл, уровень эритропоэтина ниже нормы. В мазке периферической крови – клетки-предшественники эритроцитов. Какое состояние вероятнее всего у пациента?
Полицитемия
Вторичный эритроцитоз
Артериальная гипертензия
Тромбоцитемия
Почечно-клеточный рак
84.7-летняя девочка с жалобами на скудное безболезненное кровотечение из влагалища, появившееся 6 часов назад. У ее старшей сестры первая меснтруация в 11 лет. На 95-м процентиле по весу. Кожа жирная, редкие подмышечные волосы. Молочные железы и ареолы выступают в виде конуса, без границы между ними. Наружные половые органы нормальные. Сывороточная глюкоза 189 мг/дл. Проба с диферилином – увеличение уровня лютеинизирующего гормона.
Что является основной причиной данного состояния?
Высвобождение ГнРГ (гонадотропин-рилизинг-гормон)
Переизбыток кортизола надпочечников
Эктопическая выработка гормонов
Дефицит гормонов щитовидной железы
Гиперинсулинемия
86.50-летняя женщина обратилась по поводу сердцебиения, раздражительности. За последние 4 месяца было несколько эпизодов аритмии от 30 сек-до нескольких часов. Потеря в весе 8.8 кг за последние 4 мес. В анамнезе астма, принимает ингаляционные бронходилататоры. PS 102/мин, АД 148/98 мм рт ст. Кожа теплая и влажная. В передней части шеи – пальпируется образование. Анти-ТТГ – более 1.75 МЕ/л.
Какая тактика целесообразна в долгосрочной преспективе?
Терапия L-тироксином
Терапия йодидом калия
Эстрогенная заместительная терапия
Тиреоидэктомия
Терапия пропанололом
87.Мужчина 48 лет, жалобы на кашель, невыраженные симптомы интоксикации, температура тела 37,3°С, ЧДД 18 в минуту, на рентгенограмме легочная инфильтрация в пределах 1 сегмента, лейкоциты 9,0-10,0 х 109/л. Живет в благоустроенной квартире, с женой. Врачом был открыт стационар на дому.
Какая форма заполняется на больного при открытии стационара на дому (Приказ МЗ №669 от 17.08.2015г. «Об утверждении Правил оказания стационарозамещающей помощи»)?
№ 003-2/у
№ 025-8/у
№ 030-6/у
№ 027/у
№ 066-4/у
88.Мужчина 48 лет, получил стационарозамещающую помощь по поводу внебольничной пневмонии легкого течения.
По какой форме заполняется статистическая карта при выписке пациента, получившего стационарозамещающую помощь (Приказ МЗ №669 от 17.08.2015г. «Об утверждении Правил оказания стационарозамещающей помощи»)?
№ 066-4/у
№ 003-2/у
№ 025-8/у
№ 030-6/у
№ 027/у
90.У больной А., 60 лет, в анализах периферической крови выявлены следующие изменения: Нв 78г/л., ЦП 1,3, эритроциты 2,3х1012 /л, макроцитоз. В анамнезе: много лет страдает хроническим заболеванием желудка и кишечника. Какое лечение необходимо назначить больному с данным заболеванием?
Инъекции цианокобаламина
Переливание эритроцитарной массы
Инъекции феркайла
Прием сульфата железа
Прием аскорбиновой кислоты
91.На приеме 4-х летний ребенок с жалобами на повышение температуры тела в течение 20 дней до субфебрильных цифр, постоянный кашель, снижение аппетита. В домашних условиях проводилась антибактериальная терапия препаратами пенициллина. Объективно: умеренная одышка, кожные покровы бледные с «мраморным рисунком», периоральный цианоз, в легких выслушиваются средне- и мелкопузырчатые хрипы в нижних отделах легких с обеих сторон. На рентгенограмме грудной клетки: неоднородная инфильтрация легочных полей – «снежная буря», выраженное усиление легочного рисунка за счет интерстициального и сосудистого компонентов. В ОАК - лейкоцитоз, умеренный нейтрофилез, эозинофилия, ускоренная СОЭ, анемия I степени. Какой из возбудителей имет этиологическое значение?
микоплазма
синегнойная палочка
стафилококк
стрептококк
пневмококк
92.У больного после физической нагрузки появился приступ одышки, сопровождаемый удушьем, кашлем с выделением пенистой розовой мокроты. При осмотре: в легких влажные разнокалиберные хрипы с обеих сторон, мерцательная аритмия, увеличение печени, отеки на нижних конечностях. Для какой патологии характерны указанные симптомы?
острая левожелудочковая недостаточность
приступ бронхиальной астмы
тромбоэмболия легочной артерии
спонтанный пневмоторакс
инфарктная пневмония.
94.На приеме ребенок 3 лет с жалобами родителей на отставание в физическом развитии. Объективно: нервно-психическое развитие соответствует возрастной норме, отмечается чрезмерное развитие плечевого пояса. Нижняя часть тела, конечности слабые, мышечная гипотония. Границы сердца расширены влево на 2 см. Сердечный толчок усиленный, 1 тон усиленный. Систолический шум во 2 межреберье справа. Печень не увеличена. Какой из клинических признаков наиболее достоверно подтверждает коарктацию аорты?
АД выше на руках, чем на ногах
гипертрофия миокарда правого желудочка
АД выше на ногах чем на руках
АД на руках одинаково с АД на ногах
усиление легочного рисунка на рентгенограмме
95.На приеме у врача общей практики больная девочка 9 лет с жалобами на длительный субфебрилитет, слабость и утомляемость, плохой аппетит. Из анамнеза известно, что жалобы появились после экстракции зуба 4 недели назад. Проводилась симптоматическая жаропонижающая терапия без положительного эффекта. Объективно: общее состояние тяжелое. Отмечается вялость, бледность, одышка. Пальпаторно: усиление верхушечного толчка, систолическое дрожание. Перкуторно: расширение границ сердца. Аускультативно: грубый скребущий систолический шум, связанный с I тоном, проводящийся на всю область сердца. АД 115/40 мм.рт.ст. ЭКГ: синусовая тахикардия, нормальное положение ЭОС, признаки перегрузки обоих желудочков. Предварительный диагноз?
Острая ревматическая лихорадка
Инфекционный эндокардит
Вирусный миокардит
Миокардиодистрофия
Кардиомиопатия
96.При осмотре у больного бледные кожные покровы покрытые потом. Шейные вены набухшие. АД - 180/100 мм.рт.ст, ЧСС 90 уд. в 1 мин., пульс асимметричный. Определяется расширение сосудистого пучка. По ходу аорты выслушивается систолический шум. При пальпации живот мягкий, в мезогастрии определяется пульсирующее образование. Экспресс-тест с тропонином (-). На ЭКГ признаки коронарной недостаточности, очаговые изменения миокарда, гипертрофия левого желудочка. Укажите вероятную причину имеющегося симптомокомплекса у больного.
расслаивающая аневризма аорты
гипертоническая болезнь
нестабильная стенокардия
ТЭЛА
гипертонический криз
97.Больной 55 лет с жалобами на внезапно развившуюся одышку. В анамнезе перенесенный тромбофлебит нижних конечностей. При осмотре цианоз лица и верхней половины туловища. Шейные вены набухшие, пульсирующие. В легких ослабленное везикулярное дыхание, мелкопузырчатые влажные хрипы на ограниченном участке. ЧДД – 25 в мин. Границы сердца расширены вправо. Определяется эпигастральная пульсация, усиливающаяся на вдохе. Аускультативно: акцент 2 тона над легочной артерией. АД 90/60 мм рт.ст. ЧСС 100 в мин. Укажите наиболее вероятные изменения на ЭКГ в данной ситуации:
признаки перегрузки правых отделов S1, QIII, TIII
патологические зубцы Q, подъем сегмента ST, инверсия зубца Т
подъем сегмента ST во многих отведениях
отклонение ЭОС влево, гипертрофия левого желудочка
снижение сегмента ST
98.У больного находящегося на ЭКГ-мониторном наблюдении развилась внезапная потеря сознания. Зрачки расширены. Кожные покровы бледно – серой окраски. Пульс на сонных артериях и дыхание отсутствуют. На ЭКГ хаотичные, нерегулярные, резко деформированные, различной высоты, ширины и формы волны, высокоамплитудные фибрилляции с частотой около 600 в минуту. Укажите наиболее вероятную причину данного сотояния у больного:
Фибрилляция желудочков
Трепетание желудочков
Желудочковые аритмии
Желудочковая тахикардия
Мерцательная аритмия
99.Больная 80 лет обратилась в клинику с диагнозом: острый задний инфаркт миокарда. За время наблюдения у больной периодически возникают эпизоды потери сознания с эпилептиформными судорогами и дыханием типа Чейн-Стокса. На ЭКГ: зубцы Р не связаны с комплексами QRS жестким интервалом, продолжительность интервала Р-Р = 0,80 с, R-R == 1,5 к. ЧСС 35 ударов в мин. Предположительный диагноз?
синдром Морганьи-Адамс-Стокса
эпилепсия
брадикардитическая форма мерцания предсердий
фибрилляция желудочков
синусовая брадикардия
100.У 19-летнего юноши обнаружено желтушное окрашивание склер и кожи; размеры печени по Курлову 10х9х8 см, размеры селезенки 8х6 см. Нв – 135/л. общий билирубин крови – 87 ммоль/л, непрямой – 56 ммоль/л. Уровни АЛТ и АСТ без изменений.
Какой диагноз наиболее вероятен:
синдром Жильбера
первичный билиарный цирроз печени
наследственный микросфероцитоз
хронический активный гепатит
синдром Дабина-Джонсона


Достарыңызбен бөлісу:
  1   2   3   4   5   6   7   8   9   ...   55




©engime.org 2024
әкімшілігінің қараңыз

    Басты бет